PDA

توجه ! این یک نسخه آرشیو شده میباشد و در این حالت شما عکسی را مشاهده نمیکنید برای مشاهده کامل متن و عکسها بر روی لینک مقابل کلیک کنید : طرح مسائل المپيادي



صفحه ها : [1] 2

mehrjoo
10-06-2011, 10:21 PM
سلام به همه اعضا گرامي
در اين تاپيك قصد داريم با طرح سوالات المپيادي ، قوه تحليل داده و محاسبات علاقمندان به شركت در المپياد رو يك محكي بزنيم.
به اين ترتيب كه سوالاتي در حد مرحله اول يا دوم المپياد مطرح ميشه و اعضا بايد اون مسائل رو حل كنند يا براي حلش ايده بدهند .
پس از يك يا دو هفته اگر جواب درست داده نشد ، راه حل مسئله اعلام خواهد شد .
ضمنآ از ساير مديران فروم و مدرسان المپياد هم دعوت مي كنم كه در طرح سوالات در تاپيك همكاري كنند .

---------------------------------------------------------

خب به عنوان اولين سوال ، يه مسئله جالب در حيطه نجوم كروي مطرح میکنم تا روی اون فکر کنید و براي حلش ايده بديد :

فرض کنید دوست خوبم آقای مصطفی امام مدیر سایت آوا برای تعطیلات آخر هفته یه تریپ به جزایر قناری رفته و نیمه شب در حالی که برای عکاسی در نزدیکی ساحل به سر می برند تصمیم میگیرند که گودالی به قطر 50 متر و عمق 1 کیلومتر حفر کنند و از ته گودال آسمان را رصد کنند. با فرض یکسان پراکنده بودن ستارگان در آسمان، آقا مصطفی چند درصد از کل آسمان را می تواند مشاهده کند؟

space
10-06-2011, 10:32 PM
من زیاد نجوم کروی نمیدونم
ولی فکر میکنم باید تانژانت 25 روی 1 کیلومتر حساب شه که ببینمیم اصلا چشم میتونه تفیک کنه یا نه
فقط به ذهنم رسید نمیدونم درسته یا نه

fairytale
10-06-2011, 11:19 PM
سلام به علاقه مندان نجوم کروی

یه سوال جالب مطرح میکنم تا روی اون فکر کنید و ایده بدید. اگه حلش کنید که چه بهتر:

فرض کنید دوست خوبم آقای مصطفی امام مدیر سایت آوا برای تعطیلات آخر هفته یه تریپ به جزایر قناری رفته و نیمه شب در حالی که برای عکاسی در نزدیکی ساحل به سر می برند تصمیم میگیرند که گودالی به قطر 50 متر و عمق 1 کیلومتر حفر کنند و از ته گودال آسمان را رصد کنند. با فرض یکسان پراکنده بودن ستارگان در آسمان، آقا مصطفی چند درصد از کل آسمان را می تواند مشاهده کند؟
فکر می کنم چیزی حدود 3.18 درصد می شه! اگه جوابم درسته بگین تا راهشم بگم!

MOON
10-07-2011, 01:34 PM
ممنون جناب مهرجو

وقتي ايشان ته اين گودال باشند در واقع انتهاي استوانه اي هستند به طول 1000 كيلومتر و عرض 50 متر

اگر درست وسط گودال باشند بالاي سرشان 2 مثلث قائمه وجود دارد به ارتفاع 1000 متر و قاعده 25 متر

تانژانت زاويه ديد ( آلفا ) = 50/1000

tan Alfa = 0.05

زاويه ديد = 2.8 درجه

يعني از كل آسمان 2.8 درجه مي بينند كه اين مقدار به درصد ميشود 1.5 درصد از كل يك نيم كره آسمان

پیمان اکبرنیا
10-07-2011, 01:53 PM
فکر کنم مدیران حق جواب دادن ندارند نه؟ :grin:

پس بنده یک راهنمایی میکنم: MOON زاویه بالای سر را به درستی محاسبه کردند ولی برای محاسبه درصد ستارگان باید مساحت قسمتی از آسمان که قابل مشاهده است را به مساحت کل آسمان حساب کرد. کسی بلده مساحت یک عرقچین در کره را محاسبه کنه؟ (میدانید عرقچین چیست؟)

MOON
10-07-2011, 02:02 PM
خيلي ممنون آقاي اكبرنيا
خب اگر ما بدانيم 2.8 درجه از كل 180 درجه را مي بينيم نمي توانيم نسبت ببنديم كه چند درصد مي شود ؟ ( 2.8 درجه از 180 درجه = x درصد از 100 درصد )

بعدش عرقچين ميدانم چيست اما روش محاسبه مساحت آن نه و ديگه اينكه اصلآ مساحت اين عرقچين يا كل آسمان بر اساس چه واحدي است ؟ درجه مربع مثلآ ؟؟ چه واحدي ؟؟

MOON
10-07-2011, 02:05 PM
بعدش هم آيا بايد درصد ديد از يك نيمكره را حساب كرد يا كل كره آسمان ؟

پیمان اکبرنیا
10-07-2011, 02:22 PM
خيلي ممنون آقاي اكبرنيا
خب اگر ما بدانيم 2.8 درجه از كل 180 درجه را مي بينيم نمي توانيم نسبت ببنديم كه چند درصد مي شود ؟ ( 2.8 درجه از 180 درجه = x درصد از 100 درصد )

بعدش عرقچين ميدانم چيست اما روش محاسبه مساحت آن نه و ديگه اينكه اصلآ مساحت اين عرقچين يا كل آسمان بر اساس چه واحدي است ؟ درجه مربع مثلآ ؟؟ چه واحدي ؟؟

در مورد سوال اول: نه نمیتوانیم نسبت ببندیم زیرا تابع مساحت بر حسب زاویه خطی نیست. یعنی مثلا اگر زاویه از 2.8 درجه 2 برابر بشه، مساحت عرقچین 2 برابر نمیشه.
سوال دوم: روش محاسبه مساحت عرقچین به کمک انتگرالگیری روی سطح کره است. مساحت روی کره را میتوان بر حسب زاویه فضایی بیان کرد. البته میشه برای کره شعاع دلخواهی در نظر گرفت و مساحت را با واحد مترمربع حساب کرد و بعد چون میخواهیم نسبت بگیریم مشکلی به وجود نمی آید.

سوال آخر: صورت سوال گفته نسبت به کل آسمان حساب کنید.

MOON
10-07-2011, 04:44 PM
جناب استاد اكبرنياي عزيز
ممنون از راهنمايي خوبتون
آيا ممكن است فرمول مساحت عرقچين را كه به آن اشاره داشتيد دقيق تر بنويسيد
فكر ميكنم به اشتباهم در حل بسياري از مسائل اين مدلي پي بردم
واقعآ از شما ممنونم

پیمان اکبرنیا
10-07-2011, 06:13 PM
جناب استاد اكبرنياي عزيز
ممنون از راهنمايي خوبتون
آيا ممكن است فرمول مساحت عرقچين را كه به آن اشاره داشتيد دقيق تر بنويسيد
فكر ميكنم به اشتباهم در حل بسياري از مسائل اين مدلي پي بردم
واقعآ از شما ممنونم

با توجه به این که امکان نوشتن ریاضی اینجا وجود نداره لینکی از وبلاگ آقای موسوی میگذارم(که با سرچ پیدا کردم)

فرمول مساحت عرقچین را نوشتند. فقط شما لطف کنید و فرآیند تبدیل اون زاویه 2.8 درجه به اندازه h را انجام دهید. شعاع کره را 1 فرض کنید(برای راحتی).

http://valobessin.blogfa.com/post-16.aspx

fairytale
10-07-2011, 11:39 PM
هیچ کس جواب منو نداد ولی خودم فهمیدم که جوابم غلط بود! ولی یه سوال: moon عزیز چرا ناظرو وسط گودال فرض کردین؟ من فکر می کنم بیشترین زاویه دید وقتی بوجود میاد که ناظرو در اطراف گودال (مشخصا دو سر یه قطر) فرض کنیم!

پیمان اکبرنیا
10-07-2011, 11:52 PM
هیچ کس جواب منو نداد ولی خودم فهمیدم که جوابم غلط بود! ولی یه سوال: moon عزیز چرا ناظرو وسط گودال فرض کردین؟ من فکر می کنم بیشترین زاویه دید وقتی بوجود میاد که ناظرو در اطراف گودال (مشخصا دو سر یه قطر) فرض کنیم!

بله درست میفرمایید در صورتی که ناظر بتواند ته گودال جابه جا شود مساحت بیشتری از آسمان را میبیند. یعنی اندازه عرقچین بزرگتر میشود.

mehrjoo
10-11-2011, 12:01 AM
ممنون از پیمان عزیز بابت همکاری با من در هدایت این تاپیک. تقریبا میتونم بگم که پیمان راه حل رو کامل اشاره کرده ، کافیه زاویه فضایی قابل مشاهده از مخروط ایجاد شده در کف گودال رو به زاویه فضایی کل کره که مقدارش .... هست تقسم کنید.
مشابه این سوال اگر اشتباه نکنم در سال 82 یا 83 در آزمون المپیاد نجوم سمپاد مطرح شده بود. البته اون سوال بیشتر به نظرم اطلاعات نجومی نیاز داشت و یه تخمین زدن.
حالا کسی میدونه اون سوال چی بوده ؟؟؟ (دو نقطه دی)

fairytale
10-12-2011, 11:29 PM
آقای اکبرنیا شما یه جا گفتین که برای راحتی کار شعاع کره رو یک درنظر بگیریم! ولی آخه نسبت نهایی مگه نمیشه *2pRh/4pR^2؟ پس یه R تو مخرج هست که تو درصد نهایی تاثیر میذاره بنابراین نمیشه R رو یک در نظر گرفت! پس R رو چند درنظر بگیریم؟
* منظور از p همان عدد پی و از 2^، به توان 2 می باشد!

Mostafa
10-15-2011, 12:56 AM
با اجازه از مدير محترم تالار المپياد ، سوال بعدي را من طرح مي كنم:

من عكس زير را هفته قبل در شب بارش اژدهايي در پلور گرفتم اما براي اينكه كمي مسآله جالب تر شود در تاريخ مسآله تغيير مختصري ايجاد ميكنم .

اطلاعات مسآله :

اين عكس در 20 آبان ماه در پلور ( عرض جغرافيايي 36 درجه ) گرفته شده است :


http://up3.iranblog.com/images/b6af7p5wzn5p14hwmr0n.jpg (http://up3.iranblog.com/images/b6af7p5wzn5p14hwmr0n.jpg)

محل طلوع خورشيد در صبح روز بعد را با يك فلش مشخص كنيد

چند نكته :

فرض كنيد اين عكس را پرينت كرده اند و به شما داده اند و هر بلايي بخواهيد مي توانيد سر عكس بياوريد !

نوري كه در افق شرقي هست ، آلودگي نوري است و ربطي به طلوع خورشيد ندارد .

ضمنآ از اعوجاج احتمالي در عكس صرف نظر مي كنيم .

موفق باشيد :Psmiley:

پیمان اکبرنیا
10-15-2011, 06:49 PM
آقای اکبرنیا شما یه جا گفتین که برای راحتی کار شعاع کره رو یک درنظر بگیریم! ولی آخه نسبت نهایی مگه نمیشه *2pRh/4pR^2؟ پس یه R تو مخرج هست که تو درصد نهایی تاثیر میذاره بنابراین نمیشه R رو یک در نظر گرفت! پس R رو چند درنظر بگیریم؟
* منظور از p همان عدد پی و از 2^، به توان 2 می باشد!

سلام.
درسته که یک R ساده نمیشه ولی اون h در معادله داره یک جورایی همون کار R دوم را انجام میده چون خود h درصدی از R هست و میتونه ساده بشه.

Mostafa
10-15-2011, 11:19 PM
با اجازه از مدير محترم تالار المپياد ، سوال بعدي را من طرح مي كنم:

من عكس زير را هفته قبل در شب بارش اژدهايي در پلور گرفتم اما براي اينكه كمي مسآله جالب تر شود در تاريخ مسآله تغيير مختصري ايجاد ميكنم .

اطلاعات مسآله :

اين عكس در 20 آبان ماه در پلور ( عرض جغرافيايي 36 درجه ) گرفته شده است :


http://up3.iranblog.com/images/b6af7p5wzn5p14hwmr0n.jpg (http://up3.iranblog.com/images/b6af7p5wzn5p14hwmr0n.jpg)

محل طلوع خورشيد در صبح روز بعد را با يك فلش مشخص كنيد

چند نكته :

فرض كنيد اين عكس را پرينت كرده اند و به شما داده اند و هر بلايي بخواهيد مي توانيد سر عكس بياوريد !

نوري كه در افق شرقي هست ، آلودگي نوري است و ربطي به طلوع خورشيد ندارد .

ضمنآ از اعوجاج احتمالي در عكس صرف نظر مي كنيم .

موفق باشيد :Psmiley:

نبود ؟؟؟؟؟؟؟؟؟؟؟؟؟؟؟؟؟؟؟

خيلي آسونه ها !

يه ايده اي چيزي براي حلش ندارين ؟؟؟؟؟؟؟؟؟؟؟

پیمان اکبرنیا
10-15-2011, 11:56 PM
یک راهنمایی: استوای سماوی تقریبا از ستاره وسط کمربند صورت فلکی جبار میگذره (دقیقش را در نقشه آسمان ببینید)

Amin-Mehraji
10-16-2011, 12:09 AM
http://s2.picofile.com/file/7159904080/b6af7p5wzn5p14hwmr0n.jpg (http://s2.picofile.com/file/7159904080/b6af7p5wzn5p14hwmr0n.jpg)

Mostafa
10-16-2011, 12:10 AM
ضمن تشكر از پيمان عزيز ، سعي ميكنم با چند كلمه راهنمايي بيشتري كنم :

تاريخ

ميل خورشيد

سمت طلوع

ستاره قطبي

استواي سماوي ( كمربند جبار ) .....


تقريبآ حل شد !

fairytale
10-19-2011, 01:05 PM
من مسئله اولو اینجوری حل کردم! لطف میکنین ببینین درسته یا نه؟
http://1.iped.comule.com/images/b4e9aa703d0b.jpg (http://1.iped.comule.com/images/b4e9aa703d0b.jpg)

galaxy 1996
10-19-2011, 05:35 PM
آقا لطفا در سطح پایین هم صحبت کنید!
برا کسایی که میخوا تازه شروع کنن برنامه بدید لطفا!
چه کتابی بخونن یا تنها بخونن یا با معلم و این جور حرف ها.
بازم خیلی ممنون استااااد!

Astronomer
10-19-2011, 05:44 PM
من مسئله اولو اینجوری حل کردم! لطف میکنین ببینین درسته یا نه؟
http://1.iped.comule.com/images/b4e9aa703d0b.jpg (http://1.iped.comule.com/images/b4e9aa703d0b.jpg)
چون شخص(اقاي مصطفي امام) ميتونه براي افزايش ديد خودش به كنار ديواره هاي چاه هم بره و از اونجا هم آسمون رو رصد كنه و اينكه اين هم متقارنه چون چاه رو دايره فرض ميكنيم؛ در حل قسمت اول بايد 50هزارم بگيريد نه 25هزارم...
سپس بايد بگيد كه مساحت عرقچين متناظر با قطع مخروطي با زاويه انحراف a و كره ميشه اون فرمول كه h ميشه(r(1-cos a و از اين راه يك زاويه فضايي بدست مياد كه(2pi(1-cos a است و وقتي اون رو تقسيم بر زاويه فضايي كل كره يعني 4pi جواب مسئله پيدا ميشه كه فكر ميكنم با اين جواب بايد فرض داشته باشد...

galaxy 1996
10-19-2011, 08:18 PM
باز من میگم باز حرف خودشون رو میزنن!
میگم سطح پایین!
رهنمایی!
چه کتابی بخونیم با کی بخونیم!

Sunrise
10-19-2011, 10:17 PM
باز من میگم باز حرف خودشون رو میزنن!
میگم سطح پایین!
رهنمایی!
چه کتابی بخونیم با کی بخونیم!

بابا جان :grin:
فرزندم
خب یه بار روی تالار اصلی المپیاد کلیک کنید و یه سر به تاپیک هاش بزنید...:stupido:

اولش هم هست
این اواسطشه! چه اصراری دارید از اینجا شروع کنید خب:banghead: ;)

mehrjoo
10-21-2011, 10:01 PM
آقا لطفا در سطح پایین هم صحبت کنید!
برا کسایی که میخوا تازه شروع کنن برنامه بدید لطفا!
چه کتابی بخونن یا تنها بخونن یا با معلم و این جور حرف ها.
بازم خیلی ممنون استااااد!

دوست عزیز
جهت آشنایی با منابع لطفا سایر تاپیک های مربوط به تالار المپیاد نجوم را مرور کنید.

Mostafa
10-27-2011, 06:05 PM
خب ...

برای خاک نخوردن این تاپیک و راه افتادن بچه های المپیادی سوال نسبتآ ساده ای رو طرح می کنم :

( لطفآ به اعلام پاسخ نهایی بسنده نکنید و روش حل را هم توضیح دهید )

سوال :

دنباله داری در پرنور ترین حالت در فاصله 2AU از خورشید قرار دارد . اگر خروج از مرکز مدار این دنباله دار e= 0.65 باشد ، دوره تناوب این دنباله دار را محاسبه کنید .

solh
10-27-2011, 06:33 PM
خب ...

برای خاک نخوردن این تاپیک و راه افتادن بچه های المپیادی سوال نسبتآ ساده ای رو طرح می کنم :

( لطفآ به اعلام پاسخ نهایی بسنده نکنید و روش حل را هم توضیح دهید )

سوال :

دنباله داری در پرنور ترین حالت در فاصله 2AU از خورشید قرار دارد . اگر خروج از مرکز مدار این دنباله دار e= 0.65 باشد ، دوره تناوب این دنباله دار را محاسبه کنید .

اگر منظور این باشه که دنباله دار در حضیض است ، آنگاه با تقسیم این مقدار ( 2au) بر یک منهای eمقدار نیم قطر اطول بدست میاد ، بعد با جایگذاری در قانون دوره تناوب جواب بدست میاد . احتمالا :13.66سال

Mostafa
10-29-2011, 08:56 PM
اگر منظور این باشه که دنباله دار در حضیض است ، آنگاه با تقسیم این مقدار ( 2au) بر یک منهای eمقدار نیم قطر اطول بدست میاد ، بعد با جایگذاری در قانون دوره تناوب جواب بدست میاد . احتمالا :13.66سال

پاسخ نهايي شما درست است .

اما لطفآ روش دقيق حل را هم جهت استفاده اعضا اعلام بفرماييد .

solh
10-30-2011, 10:24 PM
پاسخ نهايي شما درست است .

اما لطفآ روش دقيق حل را هم جهت استفاده اعضا اعلام بفرماييد .

2712

به خاطر این موضوع که دنباله دار در حالت پر نور تری نسبت به بقیه شرایطش قرار دارد طبیعی است که فرض کنیم دنباله دار در فاصله حد اقل از خورشید است . و همانطور که از بیضی پیداست این مینیمم برابر( a(1-e میباشد ، پس حال که این مقدار و خروج از مرکز را داریم aرا حساب میکنیم . حال از قانون نسبی دوره تناوب و نیم قطر اطول استفاده میکنیم و عناصر دنباله دار را که نسبت به زمین در نظر بگیریم ، آنگاه نسبت با جذر گرفتن از توان سوم aبدست می آید (نسبت دوره تناوب ) که اگه درست باشه برابر میشه با حدودا 13.66که چون نسبت به زمینه میشه : 13.66 سال

پیمان اکبرنیا
10-30-2011, 10:55 PM
حال من یک سوال مطرح میکنم که دوستان روش فکر کنند:

برای ناظری در شهر تهران، چند سال دیگر ستاره ی قطبی کنونی(آلفا دب اصغر)، تبدیل به ستاره ای می شود که در آسمان طلوع و غروب دارد؟ (یعنی دیگر حتی دور قطبی هم نیست).

یک مقدار فکر کنید. در صورت نیاز راهنمایی می کنم.

sooshans
10-31-2011, 07:55 PM
حال من یک سوال مطرح میکنم که دوستان روش فکر کنند:

برای ناظری در شهر تهران، چند سال دیگر ستاره ی قطبی کنونی(آلفا دب اصغر)، تبدیل به ستاره ای می شود که در آسمان طلوع و غروب دارد؟ (یعنی دیگر حتی دور قطبی هم نیست).

یک مقدار فکر کنید. در صورت نیاز راهنمایی می کنم.
سلام من بهش فکر کردم ولی از جوابم مطمئن نیستم راهمو میگم اگه درست بود بگین تا جواب نهایی رو بذارم:
خب شرط اینکه یه ستاره دور قطبی نباشه اینه که میلش از متمم عرض جغرافیایی ناظر کمتر باشه. عرض جغرافیایی تهران هم که حدودا 35.5 پس میل ستارمون باید از 54.5 کمتر بشه ما باید شرایط حدی رو بررسی کنیم پس ما باید زمانی که طول میکشه تا میل ستاره بشه از 90 به 54.5 برسه رو حساب کنیم
حالا باید حرکت تقدیمی رو بررسی کنیم من یه کم با این جای راه حلم مشکل دارم شما بگین میشه اینکارو کرد یا نه؟
ما به جای اینکه زمین رو در حال چرخش(شبیه فرفره) در نظر بگیریم ثابت نگهش میداریم و سرعتشو میدیم به ستاره هایی که بالا سرش میچرخن(چه کودکانه:hope my fake smile) اینجوری الان ما یه زمین داریم یه محور زمین و یه دایره صغیره روی کره سماوی که نقطه مورد نظر ما روی اون دایره(ستاره قطبی فعلی) الان مماس به نقطه تقاطع محور زمین و کره سماویه:you're kidding, rig
قطر این دایره که در راستای یکی از نصف النهار های کره سماویه 47 درجس چون الان ستاره قطبی 23.5 درجه انحراف داره پس اون طرف دایره با این طرفش 47 درجه تفاوت داره خب پس پایین ترین نقطه اون دایره 43 درجه میل داره ما میل 54.5 رو میخوایم با یه نسبت مثلثاتی تو اون دایره می تونیم میزان درجه ای که ستاره باید طی کنه تا میلش 54.5 بشه رو بدست میاریم بعد چون میدونیم یه دور کامل این دایره حدود 27000 سال طول میکشه می توین اون زمانو بدست بیاریم:you're kidding, rig
تا حالا اینقد پشت سر هم تایپ نکرده بودم!!!!

پیمان اکبرنیا
10-31-2011, 08:18 PM
سلام من بهش فکر کردم ولی از جوابم مطمئن نیستم راهمو میگم اگه درست بود بگین تا جواب نهایی رو بذارم:
خب شرط اینکه یه ستاره دور قطبی نباشه اینه که میلش از متمم عرض جغرافیایی ناظر کمتر باشه. عرض جغرافیایی تهران هم که حدودا 35.5 پس میل ستارمون باید از 54.5 کمتر بشه ما باید شرایط حدی رو بررسی کنیم پس ما باید زمانی که طول میکشه تا میل ستاره بشه از 90 به 54.5 برسه رو حساب کنیم
حالا باید حرکت تقدیمی رو بررسی کنیم من یه کم با این جای راه حلم مشکل دارم شما بگین میشه اینکارو کرد یا نه؟
ما به جای اینکه زمین رو در حال چرخش(شبیه فرفره) در نظر بگیریم ثابت نگهش میداریم و سرعتشو میدیم به ستاره هایی که بالا سرش میچرخن(چه کودکانه:hope my fake smile) اینجوری الان ما یه زمین داریم یه محور زمین و یه دایره صغیره روی کره سماوی که نقطه مورد نظر ما روی اون دایره(ستاره قطبی فعلی) الان مماس به نقطه تقاطع محور زمین و کره سماویه:you're kidding, rig
قطر این دایره که در راستای یکی از نصف النهار های کره سماویه 47 درجس چون الان ستاره قطبی 23.5 درجه انحراف داره پس اون طرف دایره با این طرفش 47 درجه تفاوت داره خب پس پایین ترین نقطه اون دایره 43 درجه میل داره ما میل 54.5 رو میخوایم با یه نسبت مثلثاتی تو اون دایره می تونیم میزان درجه ای که ستاره باید طی کنه تا میلش 54.5 بشه رو بدست میاریم بعد چون میدونیم یه دور کامل این دایره حدود 27000 سال طول میکشه می توین اون زمانو بدست بیاریم:you're kidding, rig
تا حالا اینقد پشت سر هم تایپ نکرده بودم!!!!

سلام

راه حل شفاهی شما تقریبا درسته. البته اون اواخرشو کامل متوجه نشدم. اگر بتونید یک شکل ساده(با نرم افزار paint) تهیه کنید و روی اون توضیح بدید فکر میکنم بتونید حلش کنید. کلیاتش درسته.

sooshans
10-31-2011, 08:29 PM
سلام

راه حل شفاهی شما تقریبا درسته. البته اون اواخرشو کامل متوجه نشدم. اگر بتونید یک شکل ساده(با نرم افزار paint) تهیه کنید و روی اون توضیح بدید فکر میکنم بتونید حلش کنید. کلیاتش درسته.
برای قسمت آخر شکل بکشم؟
شکل خیلی سخته اگه تونستم دستی میکشم ولی خودم احساس میکنم قسمت آخر یه جوب زدم!

2724

http://forum.avastarco.com/forum/images/misc/pencil.png (http://forum.avastarco.com/forum/images/misc/pencil.png)

solh
11-06-2011, 04:22 PM
من هم میتونم سوال مطرح کنم ؟
اگر شخصی در طول و عرض جغرافیایی A1,B1باشد و با سرعت Vدایره ی عظیمه ای را دور بزند که با استوا زاویه ی c میسازد . آنگاه :
الف: چه مدت طول میکشد که خورشید در نگاه شخص همان ارتفاع را در آسمان داشته باشد که موقع حرکت داشته است ؟
ب: اگر در هنگام حرکت لبه ی بالایی خورشید در افق هموار ناپدید شود ، فلق برای شخص حدودا چقدر طول میکشد ؟

che
11-06-2011, 05:59 PM
من هم میتونم سوال مطرح کنم ؟
اگر شخصی در طول و عرض جغرافیایی A1,B1باشد و با سرعت Vدایره ی عظیمه ای را دور بزند که با استوا زاویه ی c میسازد . آنگاه :
الف: چه مدت طول میکشد که خورشید در نگاه شخص همان ارتفاع را در آسمان داشته باشد که موقع حرکت داشته است ؟
ب: اگر در هنگام حرکت لبه ی بالایی خورشید در افق هموار ناپدید شود ، فلق برای شخص حدودا چقدر طول میکشد ؟

سوال شما سوال جالبی نیست چون کلش بر اساس پارامتره و هیچ عددی نداره . بنابر این جوابش فقط یک فرمول ساده است نه چیز دیگه
بهتره سوالات یا عددی باشه یا اگر هم قراره پارامتری باشه یک جور اثبات رابطه باشه نه چیزی که فقط باید حفظ بشه

Ehsan
11-06-2011, 06:17 PM
من هم میتونم سوال مطرح کنم ؟
اگر شخصی در طول و عرض جغرافیایی A1,B1باشد و با سرعت Vدایره ی عظیمه ای را دور بزند که با استوا زاویه ی c میسازد . آنگاه :
الف: چه مدت طول میکشد که خورشید در نگاه شخص همان ارتفاع را در آسمان داشته باشد که موقع حرکت داشته است ؟
ب: اگر در هنگام حرکت لبه ی بالایی خورشید در افق هموار ناپدید شود ، فلق برای شخص حدودا چقدر طول میکشد ؟


سوال شما سوال جالبی نیست چون کلش بر اساس پارامتره و هیچ عددی نداره . بنابر این جوابش فقط یک فرمول ساده است نه چیز دیگه
بهتره سوالات یا عددی باشه یا اگر هم قراره پارامتری باشه یک جور اثبات رابطه باشه نه چیزی که فقط باید حفظ بشه

البته با این دیدگاهی که شما (che ) نسبت به سوال ِ ایشون ( solh ) دارید باید خیلی از سوالات ِ المپیاد رو بریزیم دور!!!!!سوال ِ ایشون اشکالاتی داره مثلا این که مشخص نشده سرعت ِ گفته شده اینقدر زیاد هست که بشه از سرعت ِ زمین صرف ِ نظر کرد یا نه (معمولا اگر ذکر نشه ، چرخش ِ زمین منظور میشه ولی باز هم ذکر کردنش خیلی بهتره و نکردنش یه اشکاله).

اما این اشکال شبیه ِ اشکالی نیست که شما وارد کردید چون در المپیاد شاید (اغلب!!!) سوالی پارامتری طرح بشه که فرمولش رو قبلا پیدا کردند و در کتابها هست و میشه حفظش کرد ولی چیزی که مهمه اینه که اولا در المپیاد راه ِ به دست ِ آوردن ِ این فرمول مهمه ( یعنی همون اثباتی که گفتید) ثانیا اگر بنا به جلوگیری از حفظ کردن ِ فرمول هستش این راه ِ حل ِ شما (عددی کردن ِ مسئله) راه ِ خوبی نیست چون فقط کافیه شخص اعداد رو در فرمول جایگذاری کنه و جواب بگیره.

solh
11-06-2011, 07:37 PM
منظورم این بود که سرعت های متفاوت را در نظر بگیریم .
به هر حال باید یه جوری در جواب نهایی واقعیت های فیزیکی مشخص رو نتیجه بگیریم .

ali77
11-19-2011, 01:03 PM
ببخشيد ولي جواب نهايي اين سوال معلوم نشده ؟

لطفآ يك قانوني بذاريد كه اگر تا يك مدتي كسي جواب درست را نداد خود طراح سوال پاسخ صحيح را براي استفاده ديگران قرار بده

solh
11-19-2011, 07:27 PM
ببخشيد ولي جواب نهايي اين سوال معلوم نشده ؟

لطفآ يك قانوني بذاريد كه اگر تا يك مدتي كسي جواب درست را نداد خود طراح سوال پاسخ صحيح را براي استفاده ديگران قرار بده
اگه بلد بودم نمیپرسیدم ، کاملا مخالفم با اون قسمت آخر صحبتت !!!!! راستی یکی نبود جواب ما رو بده ؟

یزدان بابازاده
11-27-2011, 05:15 PM
حال من یک سوال مطرح میکنم که دوستان روش فکر کنند:

برای ناظری در شهر تهران، چند سال دیگر ستاره ی قطبی کنونی(آلفا دب اصغر)، تبدیل به ستاره ای می شود که در آسمان طلوع و غروب دارد؟ (یعنی دیگر حتی دور قطبی هم نیست).

یک مقدار فکر کنید. در صورت نیاز راهنمایی می کنم.

يك راه برداري هم وجود دارد:(شايد فهميدنش براي دوستان سخت باشد ولي اگر ياد بگيريد كلا كروي ميشه آب خوردن!!)
ميشه توي دستگاه مختصات دايره الروجي يك بردار واحد با نام p در نظر گرفت كه جهت قطب استواي سماوي را نشان مي دهد و سپس بردار ديگري با همان زاويه ي تتا (در مختصات كروي ) در نظر گرفت كه مولفه ي "في " اش با wt (سرعت زاويه اي صربدر زمان) كه سرعت زاويه اي 50 ثانيه قوس بر سال است سپس اگر اين دو بردار را در هم ضرب دات كنيم مي توانيم زاويه بين اين دو بردار كه در واقع 90 منهاي ميل بردار دومي است را بدست آوريم و سپس اگر ميل را مساوي با حد ستاره هاي دور قطبي قرار دهيم مي توانيم زمان را بدست آوريم :-)

یزدان بابازاده
12-17-2011, 04:48 PM
با سلام و کسب اجازه از دیگر دوستان .
می خواستم یک سوال المپیادی جالب (البته بیشتر مکانیکه تا نجومی ولی خب مکانیک بسیار بسیار در المپیاد نجومه!!) :
فرض کنید یک سقف نیم کره ای دارید ( مانند خانه ی اسکیمو ها ) و سطح بدون اصطکاک است . اگر از بالای سقف سر بخورید در چه ارتفاعی از زمین از روی سطح سقف جدا می شوید؟ (دقت: سقف روی زمین است و راهنمایی : از روش های کار و انرژی استفاده کنید) اگه بیشتر هم کمک خواستید در خدمتم.

andromeda-s
12-18-2011, 10:16 PM
با سلام و کسب اجازه از دیگر دوستان .
می خواستم یک سوال المپیادی جالب (البته بیشتر مکانیکه تا نجومی ولی خب مکانیک بسیار بسیار در المپیاد نجومه!!) :
فرض کنید یک سقف نیم کره ای دارید ( مانند خانه ی اسکیمو ها ) و سطح بدون اصطکاک است . اگر از بالای سقف سر بخورید در چه ارتفاعی از زمین از روی سطح سقف جدا می شوید؟ (دقت: سقف روی زمین است و راهنمایی : از روش های کار و انرژی استفاده کنید) اگه بیشتر هم کمک خواستید در خدمتم.
میشه دو سوم شعاع دایره سوال مبانی فیزیک هالی دیم هست با کمی تفاوت نکتش اینه که باید نیروی عمودی تکیه گاهو صفر در نظر بگیریم

smhm
12-22-2011, 11:59 AM
برای ناظری در شهر تهران، چند سال دیگر ستاره ی قطبی کنونی(آلفا دب اصغر)، تبدیل به ستاره ای می شود که در آسمان طلوع و غروب دارد؟ (یعنی دیگر حتی دور قطبی هم نیست).

یک مقدار فکر کنید. در صورت نیاز راهنمایی می کنم.
برای اینکه ستاره ای طلوع و غروب داشته باشه باید فاصله اون تا قطب شمال سماوی حداقل برابر با عرض جغرافیایی باشه.
خوب حالا یه راه حل عملی:
یک پرگار بردارید با یک نقشه آسمان. سوزن پرگار را بگذارید روی نقطه قطب دایره البروج (این نقطه نزدیک سحابی چشم گربه در صورت فلکی تنین قرار داره) و قلم پرگار را بگذارید روی ستاره قطبی و بعد یک دایره بزنید. این دایره مسیر حرکت قطب شمال سماوی در طول 26000 سال است.
حالا یه جایی از این دایره را می خواهیم که فاصله اش تا ستاره قطبی برابر عرض جغرافیایی باشه.
برای اینکار شعاع پرگار را به اندازه عرض جغرافیایی باز می کنیم سوزن را روی ستاره قطبی میگذاریم و کمانی می زنیم تا دایره را در نقطه مورد نظر قطع کند.
حالا باید محاسبه کنید که نسبت طول کمان به کل دایره چقدر است.

Astronomy
12-22-2011, 12:36 PM
حال من یک سوال مطرح میکنم که دوستان روش فکر کنند:

برای ناظری در شهر تهران، چند سال دیگر ستاره ی قطبی کنونی(آلفا دب اصغر)، تبدیل به ستاره ای می شود که در آسمان طلوع و غروب دارد؟ (یعنی دیگر حتی دور قطبی هم نیست).

یک مقدار فکر کنید. در صورت نیاز راهنمایی می کنم.
فکر کنم بشه 3863.88 سال دیگه

mobi
12-22-2011, 08:10 PM
راه دیگه نداره.ی راه سریع تر!یه راه کوتاه تر!!

smhm
12-24-2011, 01:00 PM
من هم میتونم سوال مطرح کنم ؟
اگر شخصی در طول و عرض جغرافیایی A1,B1باشد و با سرعت Vدایره ی عظیمه ای را دور بزند که با استوا زاویه ی c میسازد . آنگاه :
الف: چه مدت طول میکشد که خورشید در نگاه شخص همان ارتفاع را در آسمان داشته باشد که موقع حرکت داشته است ؟
ب: اگر در هنگام حرکت لبه ی بالایی خورشید در افق هموار ناپدید شود ، فلق برای شخص حدودا چقدر طول میکشد ؟
به نظر من یه راه حل میتونه اینطور باشه:
ابتدا معادله حرکت را می نویسیم یعنی معادله خطی با شیب c نسبت به استوا که از نقطه A1,B1 می گذرد.
این معادله طول و عرض جغرافیایی را در هر لحظه از زمان می دهد.
بعد فرمول ارتفاع خورشید را می نویسیم. با صرفنظر از تغییرات بعد و میل خورشید، ارتفاع بستگی به طول و عرض جغرافیایی و زمان دارد. طول و عرض هم که طبق معادله قبلی به زمان بستگی دارد پس ارتفاع خورشید فقط به زمان وابسته است. این رابطه را می توان بصورت h(t نشان داد.
در مورد سوال الف باید حساب کرد که به ازای چه مقداری از t ارتفاع دوباره به مقدار اولیه می رسد. یعنی h(t)=h(0
در مورد سوال ب هم البته احتمالا منظور مدت شفق است نه فلق. باید حساب کرد که به ازای چه مقدار از t ارتفاع خورشید منفی 18 درجه می شود. h(t)=-18

یزدان بابازاده
12-27-2011, 01:46 PM
یک سوال کروی :
مصریان اعتقاد داشتند در روزی که ستاره ی شعرای یمانی با خورشید همزمان طلوع کند زمان طغیان رود نیل است . زمان این واقع را بدست آورید.
(از حرکت تقدیمی صرفنظر و مدار زمین را دایره ای فرض کنید.)

MOON
12-27-2011, 03:42 PM
یک سوال کروی :
مصریان اعتقاد داشتند در روزی که ستاره ی شعرای یمانی با خورشید همزمان طلوع کند زمان طغیان رود نیل است . زمان این واقع را بدست آورید.
(از حرکت تقدیمی صرفنظر و مدار زمین را دایره ای فرض کنید.)

ببخشید لازم نیست بعد و میل شعرای یمانی رو بدید ؟

Astronomy
12-27-2011, 04:48 PM
ببخشید لازم نیست بعد و میل شعرای یمانی رو بدید ؟
شعرای یمانی:
بعد: 6 ساعت، 45 دقیقه، 8.9 ثانیه
میل: 16- درجه، 42 دقیقه قوسی، 58 ثانیه قوسی

یزدان بابازاده
12-27-2011, 04:53 PM
ببخشید لازم نیست بعد و میل شعرای یمانی رو بدید ؟
می شد پارامتری هم حلش کرد . مهم راه حله سواله.

Ehsan
12-28-2011, 03:55 AM
می شد پارامتری هم حلش کرد . مهم راه حله سواله.

البته مسائل ِ کروی (دست ِ کم از نظر من) بهتر هستش که عددی طرح بشوند چون در اغلب ِ اوقات روابط ِ به دست آمده چندان زیبا از آب درنمیان! مثل ِ ضلع و زاویه ی ِ مقابلی که مجهول هستند!

solh
12-28-2011, 02:48 PM
البته مسائل ِ کروی (دست ِ کم از نظر من) بهتر هستش که عددی طرح بشوند چون در اغلب ِ اوقات روابط ِ به دست آمده چندان زیبا از آب درنمیان! مثل ِ ضلع و زاویه ی ِ مقابلی که مجهول هستند!
واقعا راست میگین ، امکان اشتباه هم در محاسبات پارامتری کروی خیلی زیاده ، یه اشتباه در منفی و مثبت کل معادلات را که ممکن است بسیار روی آن وقت گذاشته اید خراب کنند .

یزدان بابازاده
12-28-2011, 05:00 PM
با سلام
منظور من چیز دیگه ای بود :
سوالات کروی معمولا دسته بندی های خاصی دارند : مثلا بعضی ها مربوط به طلوع و غروب ، بعضی ها مربوط به حرکت سیارات ، بعضی ها حرکت تقدیمی و ...
هستند . و اگر روش حل هر دسته از سوالات بدیهی شود سوالات نمونه فقط در اعداد با هم تفاوت دارند . از این سوالی هم که بنده نوشتم نمونه های بسیاری طرح شده است که مثلا به جای شعرای یمانی مثلا نسر واقع فرض مساله بوده است . دوستان اگر روی مساله بطور پارامتری تسلط پیدا کنند هر سوالی در این رابطه را می توانند با یک جایگذاری ساده بدست بیارن . منظورم اینبود!

یزدان بابازاده
12-29-2011, 08:38 PM
خب حالا وقت جواب دادن به سواله برای دوستانی که همچنان در حال فکر کردن روی این سوالند عرض کنم که من برای این سوال دو روش حل یکی آسون ولی طولانی ( به اصطلاح خر کاری!) و یکی ممکن است در ابتدا کمی پیچیده به نظر برسد ولی بسیار کوتاه تر و راحت تر است که بنده راه حل دوم را می گویم و راه حل اول را هم اگه کسی خواست بگه راهنمایی می کنم!
http://www.astroupload.com/uploads/13251781521.jpg
http://www.astroupload.com/uploads/13251781522.jpg

در کره سماوی اولی دایره ابروج و استوای سماوی با هم کشیده شده اند که محل برخوردشان نقطه ی حمل است و محل برخورد دایره البروج و افق محل خورشید است . یکی از اضلاع lst-270 است که lst = H+RA یا زاویه ساعتی ستاره + بعد آن . بعد ستاره که ثابت و مشخص است و زاویه ساعتی ستاره در حال طلوع هم با رابطه ی cosH=-tan(phi)* tan (delta) بدست می آید که از حل مثلث کروی landa sun بدست می آید که خودش برابر است با n*360/365.25 که n تعداد روز های گذشته از اول فروردین است . به همین سادگی!

albertini
12-29-2011, 09:54 PM
آقاي man با تشكر از شما من در رابطه با همين نوع مسائل سوالي دارم كه جواب مشخصي در كتابها پيدا نكردم كه اگر شما يا هركدام از دوستان ديگر كمك كنند خيلي ممنون مي شوم

اگر در شهري با عرض جغرافيايي F ستاره اي با ميل m طلوع كند من رابطه پيدا كردن سمت طلوع آن را بلدم ، اما ميخواهم ببينم چطور بايد سمت و ارتفاع آنرا در N ساعت آينده بدست بياوريم ؟

ممنون از آموزشهاي خوبتان

یزدان بابازاده
12-30-2011, 08:42 AM
خواهش می کنم.
ببینید از همان راهی که شما سمت ستاره در حال طلوع یا غروب را بدست می آورید می توانید زاویه ساعتی طلوع یا غروب را نیز بدست آورید. سپس از آنجا که می دانیم در هر ساعت زاویه ساعتی 15 درجه زیاد می شود و با دانستن تعداد ساعت های زیاد شده که در این جا n ساعت است زاویه ساعتی جدید را بدست می آورید بعد با حل مثلثی pz* (پی - زد-ستاره!) که زاویه ساعتی ، میل-90 ، عرض جغرافیایی-90 را داریم و با قانون کسینوس ها ارتفاع-90 و با قانون چهارجزئی سمت غربی یا شرقی ستاره بدست می آید .

یزدان بابازاده
12-31-2011, 12:36 PM
یک سوال کروی :
ثابت کنید اگر در یک لحظه شکست جو زمین بر میل ستاره تاثیری نداشته باشد در آن لحظه سمت ستاره بیشینه است.
!

solh
12-31-2011, 10:09 PM
دوستان ، اگه میشه همه با هم همکاری کنیم تا یکی از سوالات به طور کامل حل بشه ، یا کنار گذاشته بشه ، بعد بریم سراغ بعدی !! اینطوری هم اونی که میپرسه مطمعن میشه روی سوالش فکر میکنن ، هم سوالات منظم تر میشن !!

smhm
01-01-2012, 02:06 PM
یک سوال کروی :
ثابت کنید اگر در یک لحظه شکست جو زمین بر میل ستاره تاثیری نداشته باشد در آن لحظه سمت ستاره بیشینه است.
!
می دانیم شکست جو فقط بر ارتفاع تأثیر داره و بر سمت تأثیر نداره.
پس نتیجه می گیریم که خط واصل بین قطب شمال سماوی تا ستاره (خط px) که تعیین کننده میل ستاره است باید موازی با افق باشد تا شکست جو بر آن تأثیر نداشته باشد.

3154

یزدان بابازاده
01-01-2012, 04:29 PM
کاملا درست است!
از حل معادلات شکست هم می شود به این نتیجه رسید که من به دوستان علاقه مند توصیه می کنم این کار را بکنند تا بیشتر با معادلات شکست آشنا شوند.

smhm
01-02-2012, 11:03 AM
یک سؤال هم بنده طرح می کنم البته برخلاف سوالات المپیادی که چندان کاربردی نیست این سوال یه کم کاربردیه.

دیواری را فرض کنید که در امتداد شمال-جنوب و قائم بر زمین کشیده شده است. از لحاظ دینی، زمانی که سایه دیوار محو می شود لحظه اذان ظهر است و زمانی که سایه دیوار به اندازه خود دیوار می شود زمان نماز عصر است.
حال باتوجه به داده های رصدی زیر، زمان اذان عصر را تعیین کنید:

ساعت ...... سمت خورشید ..... ارتفاع خورشید
14:00 ........... 31.4 .................. 30.2
14:10 .......... 33.8 .................. 29.1
14:20 ......... 36.1 ................... 27.9
14:30 ......... 38.2 .................. 26.6
(زاویه سمت از جنوب به سمت غرب اندازه گیری شده)

یزدان بابازاده
01-02-2012, 02:17 PM
سلام smhm عزيز . چند نكته درباره ي سوالتان:
1- زمان اذان ظهر ديوار باز هم سايه دارد چون خورشيد ميل دارد مگر اينكه دقيقا در نقطه ي سرسو اذان ظهر گفته شود
2- تا آنجا كه من مي دانم اذان عصر را زماني مي گويند كه شفق در حال تمام شدن است و آسمان رو به تاريكي كامل مي رود و در آن زمان نيز ديوار سايه اي ندارد.
3-و نكته ي ديگر اينكه منظورتان از اينكه سايه به اندازه ي خود ديوار مي شود منظور ارتفاع ، عرض ، طول يا ... كجاي ديوار است؟
لطفا اطلاعات يا راهنمايي هاي لازم را ارائه نماييد .
با تشكر!

یزدان بابازاده
01-02-2012, 02:20 PM
می دانیم شکست جو فقط بر ارتفاع تأثیر داره و بر سمت تأثیر نداره.
پس نتیجه می گیریم که خط واصل بین قطب شمال سماوی تا ستاره (خط px) که تعیین کننده میل ستاره است باید موازی با افق باشد تا شکست جو بر آن تأثیر نداشته باشد.

3154
فكر مي كنم درست تر اينست كه بگوييم :
خط واصل سرسو به ستاره موازي استواي سماوي باشد كه در نتيجه ي آن زاويه ي اتا (سرسو-ستاره-قطب سماوي) 90 درجه مي شود و سمت هم بيشينه.

یزدان بابازاده
01-03-2012, 06:20 PM
خب دوستان تا اصلاحات سوال قبلی صورت بگیره برای بیکار نموندن این تاپیک یه سوال می ذارم:
روی یک کاغذ یک دستگاه مختصات به مرکزیت محل برخورد نصف النهار ناظر و استوای سماوی بکشید بگونه ای که محور y نصف النهار ناظر و محور x استوای سماوی باشد. حال محل خورشید در ساعت 12 ظهر به وقت محلی در روز های متوالی را اگر روی این دستگاه مختصات بکشیم و نقاط را به هم وصل کنیم شکل نهایی چه می شود (حدس زدن ممنوع فقط اثبات!!D:!!)

smhm
01-04-2012, 10:11 AM
سلام smhm عزيز . چند نكته درباره ي سوالتان:
1- زمان اذان ظهر ديوار باز هم سايه دارد چون خورشيد ميل دارد مگر اينكه دقيقا در نقطه ي سرسو اذان ظهر گفته شود
2- تا آنجا كه من مي دانم اذان عصر را زماني مي گويند كه شفق در حال تمام شدن است و آسمان رو به تاريكي كامل مي رود و در آن زمان نيز ديوار سايه اي ندارد.
3-و نكته ي ديگر اينكه منظورتان از اينكه سايه به اندازه ي خود ديوار مي شود منظور ارتفاع ، عرض ، طول يا ... كجاي ديوار است؟
لطفا اطلاعات يا راهنمايي هاي لازم را ارائه نماييد .
با تشكر!
1- سایه دیوار را با سایه شاخص اشتیاه نگیرید. سایه دیوار پهنایی است که لبه آن موازی با قاعده دیوار است و منظور از طول سایه، فاصله این دو خط است. نمیدونم متوجه شدید یا نه.
زمان ظهر هم چون خورشید در سمت جنوب قرار می گیرد، سایه دیوار روی خود دیوار می افتد و لذا سایه دیوار محو می شود. این مسئله در همه جا و همه روز صادق است.
2- موقعی که شفق تمام می شود زمان نماز عشاء است نه عصر!
3- همانطور که گفتم منظور سایه لبه دیوار است نه ارتفاع دیوار. یعنی شما فرض کنید دیوار تا قطب شمال و جنوب کشیده شده و نمی تونید سایه انتهای دیوار را ببینید بلکه فقط می تونید سایه لبه دیوار را ببینید. و اندازه سایه، فاصله خط سایه تا خط پای دیوار است.

یزدان بابازاده
01-04-2012, 11:20 AM
1- سایه دیوار را با سایه شاخص اشتیاه نگیرید. سایه دیوار پهنایی است که لبه آن موازی با قاعده دیوار است و منظور از طول سایه، فاصله این دو خط است. نمیدونم متوجه شدید یا نه.
زمان ظهر هم چون خورشید در سمت جنوب قرار می گیرد، سایه دیوار روی خود دیوار می افتد و لذا سایه دیوار محو می شود. این مسئله در همه جا و همه روز صادق است.
2- موقعی که شفق تمام می شود زمان نماز عشاء است نه عصر!
3- همانطور که گفتم منظور سایه لبه دیوار است نه ارتفاع دیوار. یعنی شما فرض کنید دیوار تا قطب شمال و جنوب کشیده شده و نمی تونید سایه انتهای دیوار را ببینید بلکه فقط می تونید سایه لبه دیوار را ببینید. و اندازه سایه، فاصله خط سایه تا خط پای دیوار است.

سلام
مرسی از توضیحتان
1- بخش 1 رو گرفتم
2- مگه نماز عصر زمان داره ؟؟!
3- بخش 3 رو هم گرفتم

یزدان بابازاده
01-04-2012, 11:43 AM
خب بخشی از راه حل سوالی که smhm عزیز طرح کرده اند رو ارائه می کنم و بقیه ش رو هم به بقیه دوستان می سپرم تا بقیه هم بتنوانند روی سوال فکر کنند.
این بخش از راه حلی که ارائه می کنم به عنوان راهنمایی است پس اگه کسی می خواد خودش سوالو حل کنه این پست رو اصلا نخونه!!
: خب اول از همه در شکل های زیر منظور از a ارتفاع خورشید ، l طول هر پرتوی سایه ، A سمتی هست که smhm قرار داد کردند ، x هم فاصله ی لبه ی سایه تا دیوار و h ارتفاع دیوار است .
درباره ی شکل 1 : هر پرتوی خورشید پس از برخورد با لبه ی دیوار سایه ای ایجاد می کند .
http://www.astroupload.com/uploads/13256639021.png
شکل 2: طول پرتوی سایه به دلیل زاویه داشتن خورشید که این جا A-90 است فاصله اش با لبه ی دیوار تغییر می کند .
http://www.astroupload.com/uploads/13256639022.png
که در نهایت رابطه ی فاصله ی سایه با دیوار این گونه به سمت و ارتفاع خورشید مربوط می شود :
x=h.cot a.sin a
بقیه اش هم که می شه چند تا جاگذاری ساده و البته برای بدست آوردن زمان باید ارتفاع و سمت را به راویه ساعتی و آنرا به ساعت تبدیل کنید!

smhm
01-04-2012, 01:27 PM
درسته.
اگر ارتفاع دیوار را یک واحد در نظر بگیریم طول سایه دیوار برابر cot h. sinA می باشد. (همان فرمولی که جناب man بدست آوردند.)
حال باید طول سایه برابر با ارتفاع دیوار بشه. یعنی cot h.sinA=1

3168این مقدار را در جدول داده ها بررسی می کنیم:

cot h.sinA ........ h ........... A ......... time
14:00 ...... 31.4 ..... 30.2 ..... 8952/0
14:10 ..... 33.8 ..... 29.1 ..... 9995/0
14:20 ..... 36.1 .... 27.9 ..... 1.1128
14:30 .... 38.2 ..... 26.6 ..... 1.2349

همانطور که در جدول مشخص است مقدار طول سایه در ساعت 14:10 بسیار به یک نزدیک است و این همان وقت فضیلت نماز عصر است.

smhm
01-04-2012, 01:45 PM
جالبه بدونید: دیوار مسجد النبی(ص) در مدینه بدستور پیامبر(ص) بهمین صورت ساخته شد و وقت ظهر و عصر از روی همین دیوار تعیین می شد.
این مسئله بعدها انگیزه ای شد برای یک دانشمند ریاضیدان بنام ابوالوفای بوزجانی. تحقیقات او بر روی تأثیر ارتفاع خورشید و طول سایه منجر به کشف نسبتهای مثلثاتی از جمله ظل (تانژانت) و جیب (سینوس) شد.
بعدها وقتی کتابهای دانشمندان اسلامی در اروپا ترجمه می شد کلمه «جِیب» را به اشتباه «جَیب» با فتحه به معنی گریبان خواندند و برای معادل آن واژه سینوس به معنی گریبان را در نظر گرفتند!

یزدان بابازاده
01-04-2012, 04:14 PM
خب جواب سوال قبلی هم رو می ذارم :
جواب همان طور که احتمالا(؟!) حدس زده اید آنالما می شود برای اثبات:
ببینید زمان 12 ظهر به زمان محلی همیشه خورشید متوسط روی مرکز مختصات است و خورشید واقعی دارای x و y است .
که x تفاوت دو بعد خورشید هاست که ، بعد خورشید متوسط همان طول دایره البروجی خورشید واقعی است . و y خورشید واقعی همان میل آن است.
http://www.astroupload.com/uploads/13256802131.png
روابط زیر بدست می آیند =
sin(delta)=sin(e). sin(landa
tan(bod)=tan(landa).cos(e
از رابطه های بالا نیز می توان نمودار آنالما را کشید ولی برای این که خوش تعریف بشوند باید از روابط تقریبی استفاده کنیم که خیلی در شکلمان تاثیر نمی گذارد ولی خوب بالاخره به آنالما می رسیم!!
چون دلتا بازه ی نسبتا کمی جابجا می شود می توان آن را با خودش تقریب زد و از طرفی دلتا با y برابر است و e نیز تقریبا کوچک است و سینوس اش را با خودش تقریب می زنیم و می توان نوشت => y=e.sin(landa
bod را نیز می توانیم به شکل x-landa بنویسیم و به طور تقریبی به وسیله ی بسط تیلور cos(e و tan(landa-x را بسط می دهیم .
tan(landa)- (x/(cos(landa)^2)) = tan(landa) . (1-((e^2)/2)
می توان ساده کرد :
X=sin(landa).cos(landa). (e^2/2
و سپس نوشت :
x= sin(2landa).e^2 /4
هووف تموم شد ! لطفا خواهشا تمنا می کنم اگه هر جاییشو متوجه نشدید بگید تا توضیح بدم.

Ehsan
01-05-2012, 06:03 PM
برای ِ یک مدل ِ ساده از روشن شدن ِ آسمان (فلق و شفق) در نظر بگیرید که برای ِ روشن شدن ِ افق، قسمت ِ فوقانی جو که در افق می بینیم باید به طور ِ مستقیم توسط ِ خورشید روشن بشه. حالا با فرض ِ این که ارتفاع ِ جو، 100 کیلومتر و شعاع ِ زمین 6400 کیلومتر هستش، پیدا کنید که خورشید باید چه قدر نزدیک ِ افق باشه که فلق یا شفق اتفاق بی افته.
(اغلب عدد ِ واقعی رو 18 درجه ذکر می کنند!)

یزدان بابازاده
01-06-2012, 02:29 PM
سلام
به راه حل فکر کنم می تونه این باشه : از بالا ترین نقطه ی خورشید (لبه ی بالا) به سطح زمین مماس می کنیم بگونه ای که در نقطه ای جو را قطع کند.اگر جو بالای سر ناظر از آن نقطه عبور کند شفق یا فلق برای او تمام می شود . ( در واقع مسئله یه جوریا مسله مسائله انخفاص می مونه!)
http://www.astroupload.com/uploads/13258472351.png
http://www.astroupload.com/uploads/13258472352.png
فقط یه توضیحی بدم درباره ی شکل 2
یک خط موازی افق از نقطه ای که پرتو های خورشید به جو می خورند رسم می کنم بدلیل موازی بودن دو خط زاویه ی بین این خط و پرتوی خورشید همان زاویه ی بین افق و پرتوی خورشید می شود (همان 18 درجه معروف!) . حالا اسمشو می ذارم x و به همراه زاویه پایینیش که متممش هست 90 درجه می شه . از طرفی همون متممه با زاویه ای که داخل زمین ایجاد شده متمم اند در نتیجه زاویه توی زمین هم x است حالا با نوشتن یک کسینوس این زاویه بدست می آد.

پیمان اکبرنیا
01-06-2012, 06:26 PM
سلام
به راه حل فکر کنم می تونه این باشه : از بالا ترین نقطه ی خورشید (لبه ی بالا) به سطح زمین مماس می کنیم بگونه ای که در نقطه ای جو را قطع کند.اگر جو بالای سر ناظر از آن نقطه عبور کند شفق یا فلق برای او تمام می شود . ( در واقع مسئله یه جوریا مسله مسائله انخفاص می مونه!)
http://www.astroupload.com/uploads/13258472351.png
http://www.astroupload.com/uploads/13258472352.png
فقط یه توضیحی بدم درباره ی شکل 2
یک خط موازی افق از نقطه ای که پرتو های خورشید به جو می خورند رسم می کنم بدلیل موازی بودن دو خط زاویه ی بین این خط و پرتوی خورشید همان زاویه ی بین افق و پرتوی خورشید می شود (همان 18 درجه معروف!) . حالا اسمشو می ذارم x و به همراه زاویه پایینیش که متممش هست 90 درجه می شه . از طرفی همون متممه با زاویه ای که داخل زمین ایجاد شده متمم اند در نتیجه زاویه توی زمین هم x است حالا با نوشتن یک کسینوس این زاویه بدست می آد.

سلام

ممنون از زحمات شما در جواب دادن و رسم شکل. میگم حالا که بسیار زحمت میکشید و شکل میکشید، میتونید نرم افزاری برای ترسیم مثل corel یا autocad را یاد بگیرید که شکلهاتون هم سریعتر تموم بشه و هم واضحتر باشه.

یزدان بابازاده
01-06-2012, 07:15 PM
سلام

ممنون از زحمات شما در جواب دادن و رسم شکل. میگم حالا که بسیار زحمت میکشید و شکل میکشید، میتونید نرم افزاری برای ترسیم مثل corel یا autocad را یاد بگیرید که شکلهاتون هم سریعتر تموم بشه و هم واضحتر باشه.
سلام
خواهش می کنم.
چشم از این به بعد سعی امو می کنم شکل ها واضح تر باشه.

Ehsan
01-07-2012, 05:17 PM
سلام
به راه حل فکر کنم می تونه این باشه : از بالا ترین نقطه ی خورشید (لبه ی بالا) به سطح زمین مماس می کنیم بگونه ای که در نقطه ای جو را قطع کند.اگر جو بالای سر ناظر از آن نقطه عبور کند شفق یا فلق برای او تمام می شود . ( در واقع مسئله یه جوریا مسله مسائله انخفاص می مونه!)
http://www.astroupload.com/uploads/13258472351.png
http://www.astroupload.com/uploads/13258472352.png
فقط یه توضیحی بدم درباره ی شکل 2
یک خط موازی افق از نقطه ای که پرتو های خورشید به جو می خورند رسم می کنم بدلیل موازی بودن دو خط زاویه ی بین این خط و پرتوی خورشید همان زاویه ی بین افق و پرتوی خورشید می شود (همان 18 درجه معروف!) . حالا اسمشو می ذارم x و به همراه زاویه پایینیش که متممش هست 90 درجه می شه . از طرفی همون متممه با زاویه ای که داخل زمین ایجاد شده متمم اند در نتیجه زاویه توی زمین هم x است حالا با نوشتن یک کسینوس این زاویه بدست می آد.

این همون چیزی بود که من می خواستم! فلق اون نقطه ای از جو هستش که روشن شده. حالا دو برابر ِ زاویه ِ اون مثلثی که کشیدید می شه زاویه ای که من می گم. حالا که زحمت ِ شکل و اینها رو کشیدید زحمت ِ حساب و کتابش رو هم بکنید و نتیجه رو بگید!:wink:

یزدان بابازاده
01-07-2012, 06:52 PM
حساب کردم خود زاویه شد 10/05 و دو برابر ش هم میشه 20.1 درجه!!

یزدان بابازاده
01-08-2012, 02:19 PM
سلام بر دوستان همراه این تاپیک :
دیدم یه چند وقتیه داریم سوال های سخت ، با توضیحاتی نا مفهوم می ذاریم شما ها هم که صداتون در نمیاد خدا رو شکر!!
یه چند تا سوال مفهومی می ذارم خودتون روشون فکر کنید و براشون یک خط تا دو خط جواب تشریحی بنویسید (اینجا نه ها!!)
1- شرط دور قطبی شدن ستاره چیه؟
2-در چه عرض های جغرافیایی خورشید حداقل 24 ساعت بالای افق است؟
3- در مختصات p ثابت آیا k می چرخد؟
4- شرطی برای دمای تعادل یک جسم بدست بیارید (مثلا زمین)
5- چرا ستاره ها هر روز حدود 4 دقیقه دیرتر طلوع می کنند؟
6- چرا بیشترین کشیدگی یک سیاره داخلی زمانی است که خط دید ما بر مدارش مماس شود؟
7-سیارات خارجی هم هلال دارند؟
8- سعی کنید ثابت k فرمول سوم کپلر رو بدست بیارید (برای مدار های دایره ای )
9- سعی کنید ثابت G فرمول گرانش رو با روشی ابتکاری بدست بیارید .(نیوتون خودش اینکارو کرده بود و تا مرتبه ی -11 درست بوده !)
10- در حرک دایره ای اگر با سرعت ثابت حرکت کنیم چرا بازم شتاب داریم؟

این سوالا رو گذاشتم تا این تاپیک واسه همه ی دوستان قابل استفاده باشه . اگه فکر می کنید کار مسخره ای هست بگید تا دیگه نذارم.

Astronomy
01-08-2012, 07:14 PM
سلام بر دوستان همراه این تاپیک :
دیدم یه چند وقتیه داریم سوال های سخت ، با توضیحاتی نا مفهوم می ذاریم شما ها هم که صداتون در نمیاد خدا رو شکر!!
یه چند تا سوال مفهومی می ذارم خودتون روشون فکر کنید و براشون یک خط تا دو خط جواب تشریحی بنویسید (اینجا نه ها!!)
1- شرط دور قطبی شدن ستاره چیه؟
2-در چه عرض های جغرافیایی خورشید حداقل 24 ساعت بالای افق است؟
3- در مختصات p ثابت آیا k می چرخد؟
4- شرطی برای دمای تعادل یک جسم بدست بیارید (مثلا زمین)
5- چرا ستاره ها هر روز حدود 4 دقیقه دیرتر طلوع می کنند؟
6- چرا بیشترین کشیدگی یک سیاره داخلی زمانی است که خط دید ما بر مدارش مماس شود؟
7-سیارات خارجی هم هلال دارند؟
8- سعی کنید ثابت k فرمول سوم کپلر رو بدست بیارید (برای مدار های دایره ای )
9- سعی کنید ثابت G فرمول گرانش رو با روشی ابتکاری بدست بیارید .(نیوتون خودش اینکارو کرده بود و تا مرتبه ی -11 درست بوده !)
10- در حرک دایره ای اگر با سرعت ثابت حرکت کنیم چرا بازم شتاب داریم؟

این سوالا رو گذاشتم تا این تاپیک واسه همه ی دوستان قابل استفاده باشه . اگه فکر می کنید کار مسخره ای هست بگید تا دیگه نذارم.
1- delta>90-phi
2- عرض های 90 تا 66.55 درجه شمالی و جنوبی
3- متوجه نشدم سوال رو:stupido:
4- !!!!!!!!!!!!
5- سوال غلط است:blink:، ستاره ها هر روز 4 دقیقه زور تر طلوع می کنند
زمان گردش زمین: 23:56:04 شبانه روز: 24:00:00 اختلاف ایندو می شود 3:56 ثانیه که حدودا همون 4 دقیقه هست:t5hq2u:
6- چون باید به نقطه انتهای مدار خود برود (از دید ما) تا بیشترین فاصله ضاهری از خورشید را داشته باشد و ما که نگاه می کنیم انتهای مدار در راستای دید ما مماس است
7- خیر:mah (chilling):
8- آقا اینا کار ما نیست:hidden:

9- حوصله داریاااا:closed_2:
10- شتاب یعنی تغییراتی سرعت که سرعت برداری هست که تشکیل شده از اندازه و جهت
در حر کت دایره ای با سرعت ثابت اندازه سرعت تغییر نمی کنه ولی جهت تغییر می کنه
پس حرکت شتاب داره !! ;)
سوالا خیلی جالبه ولی فکر نمی کنم زیاد المپیادی باشه

یزدان بابازاده
01-08-2012, 07:46 PM
سلام
سوال 5 از قصد بود ! ، سوال 3 هم منظور اینه که تو مختصاتی که p ثابته آیا k طی مدت زمانی خاص حرکت می کند؟ ، سوال 7 جوابتون اشتباهه ، 8 و 9 هم روش فک کنید جالبه

Amirali
01-08-2012, 08:07 PM
سلام
سوال 7 جوابتون اشتباهه

نه اشتباه نیست سیارات خارجی هلال ندارن

یزدان بابازاده
01-08-2012, 08:11 PM
نه اشتباه نیست سیارات خارجی هلال ندارن

چرا دارن من خودم وقتی فهمیدم کلی تعجب کردم ولی دارند.

che
01-08-2012, 08:18 PM
چرا دارن من خودم وقتی فهمیدم کلی تعجب کردم ولی دارند.

ممكنه منظور شما اهله يا وضعيت باشه كه البته با هلال فرق مي كنه

سيارات خارجي مثل مريخ هلال ( به معني فاز هاي مختلف ) ندارند . اما وضعيتي مثله تربيع دارن

تربيع به زماني ميگيم كه جدايي خورشيد و سياره 90 درجه باشه اما در اين حالت هم كه اسمش تربيع هست باز هم شكل كلي سياره به صورت كامل ديده مشيه

Amirali
01-08-2012, 08:21 PM
چرا دارن من خودم وقتی فهمیدم کلی تعجب کردم ولی دارند.

نه اونیکه شما میگی اهله هستش
مثلا مریخ رو حداقل با فاز 89% میشه دید که به 89% نمیگن هلال
در مورد بقیه هم ان قدری کمه که تقریبا میگیم ندارن

پیمان اکبرنیا
01-08-2012, 08:21 PM
بستگی داره تعریفتون از هلال چی باشه؟ آیا اگر 70 درصد جسم روشن باشه، اسمش هلال هست یا نه؟

سیارات خارجی همیشه بیشتر از نصفشون روشنه، یعنی همیشه بیش از 50 درصد. مثلا درصد مساحت روشنشون عددی است که بین 80 تا 100 درصد تغییر میکنه. ( این عدد برای هر سیاره فرق داره و مثلا اگر برای مریخ بین 80 تا 100 درصد باشه، برای زحل بین 98 تا 100 تغییر میکنه).

حالا با این تعریف اونها هم هلال دارند، ولی نه یک هلال کاو، بلکه یک هلال کوژ.

در ضمن اهله، جمع عربی کلمه هلال هست و معنی دیگه ای نمیده! ;)

پی نوشت: عددها مثال بود و همینجوری نوشتمشون!

Amirali
01-08-2012, 08:30 PM
در ضمن اهله، جمع عربی کلمه هلال هست و نه چیز دیگه ای!

درسته
اما وقتی میگیم یه جرم آسمانی اهله داره منظور تغییر فاز اون جرمه
در مورد تعریف هلال هم ما که هیچ وقت به ماه تثلیث هلال نمیگیم که!

پیمان اکبرنیا
01-08-2012, 08:55 PM
درسته
اما وقتی میگیم یه جرم آسمانی اهله داره منظور تغییر فاز اون جرمه
در مورد تعریف هلال هم ما که هیچ وقت به ماه تثلیث هلال نمیگیم که!

بله اما من تا به حال ندیدم به ماهی که بیش از 50 درصدش روشن باشه در فارسی اسم خاصی نسبت بدن. بنابراین مجبوریم بگیم هلال.

در زبان انگلیسی به هلال کاو(کمتر از 50 درصد) میگن: Crescent
و به هلال کوژ(بیش از 50 درصد) میگن: Gibbous

در فارسی اما به جز هلال چیزی نشنیدم.

یزدان بابازاده
01-08-2012, 09:45 PM
سلام
منم با نظر آقای اکبرنیا موافقم ، وقتی من خودم این قضیه رو فهمیدم کلی تعجب کردم و فکر می کنم علتش هم اینه که اکثر رصد های ما از سیارات خارجی زمان پرنوریشان یعنی مقابله است که در آن هنگام هم قرص کامل سیاره دیده می شود ! برای همین است که درک درستی از اهله (هلال ، فاز اصلا هر چی !) سیارات خارجی نداریم!

یزدان بابازاده
01-11-2012, 05:18 PM
خب چون این تاپیک مملو از همراه و بازدید کننده است ! و به نظر هم از سوالات مفهومی استقبال شده بود من یه چندتا دیگه میذارم با هم حل می کنیمشون:
1- علت بیضی شدن خورشید یا ماه در افق چیه؟
2- ممکنه مسیر یه سیاره دقیقا سهمی بشه؟
3- فوتون چیست؟
4-برخورد کشسان رو توضیح دهید
5-مدول فاصله چیست؟
6-مزایای تلسکوپ شکستی چیه؟
7- بر اثر حرکت تقدمی آیا حمل جابجا می شه؟

یزدان بابازاده
01-13-2012, 06:26 PM
آقا حلال واسه این سوالا نبود ؟ من فعلا اینارو جواب نمی دم تا خودتون به جواب برسید ولی اگه کسی به جواب رسیده حتما بیاد اینجا بذاره دیگه!!

andromeda-s
01-13-2012, 08:31 PM
خب چون این تاپیک مملو از همراه و بازدید کننده است ! و به نظر هم از سوالات مفهومی استقبال شده بود من یه چندتا دیگه میذارم با هم حل می کنیمشون:
1- علت بیضی شدن خورشید یا ماه در افق چیه؟
2- ممکنه مسیر یه سیاره دقیقا سهمی بشه؟
3- فوتون چیست؟
4-برخورد کشسان رو توضیح دهید
5-مدول فاصله چیست؟
6-مزایای تلسکوپ شکستی چیه؟
7- بر اثر حرکت تقدمی آیا حمل جابجا می شه؟
1-به خاطر بیضی بودن مدار زمین و انحراف 23.5 درجش(اگر سوالو درست متوجه شده باشم)

2-طبق قوانین کپلر همه سیارات روی مدار بیضوی حرکت می کنند .فکر نمی کنم بشه.

3-بسته هایی از انرژین با جرم خیلی ناچیز

4-هنوز به این قسمت فیزیک نرسیدم ولی در این حالت اندازه حرکت خطی دو ذره ، قبل و بعد از برخورد بقا خواهد داشت و علاوه بر آن انرژی جنبشی نیز ثابت خواهد بود و ذرات بعد از برخورد متناسب با جرم خود و سرعت قبل از برخورد پراکنده می‌‌شوند. در این حالت هیچگونه نیروی تلف کننده یا غیرپاستیاری وجود ندارد.

5-به اختلاف قدر ظاهری و قدر مطلق ستاره می گن.

6-اجرامو شفاف تر نشون میده برای رصد سیاراتم بهتره .

7-بله جابه جا میشه اصلا به خاطر همینه که بهش حرکت تقدیمی اعتدالین می گن.

sooshans
01-13-2012, 09:56 PM
خب چون این تاپیک مملو از همراه و بازدید کننده است ! و به نظر هم از سوالات مفهومی استقبال شده بود من یه چندتا دیگه میذارم با هم حل می کنیمشون:
1- علت بیضی شدن خورشید یا ماه در افق چیه؟
2- ممکنه مسیر یه سیاره دقیقا سهمی بشه؟
3- فوتون چیست؟
4-برخورد کشسان رو توضیح دهید
5-مدول فاصله چیست؟
6-مزایای تلسکوپ شکستی چیه؟
7- بر اثر حرکت تقدمی آیا حمل جابجا می شه؟
1-تفاوت مقدار شکست برای ارتفاع های مختلف خورشید یا ماه
2-یه کم واضح تر لطفا! برای چی نباشه؟
3-شنیدم بسته انرژیه!
4-برخوردیه که انرژی جنبشی قبل و بعد از برخورد برای سیستم تغییر نمیکنه
5-قدر ظاهری منهای مطلق!
6-عمرش زیاده! تصویرش با کیفیته!(البته فک کنم)
7-حمل که جابجا نمیشه! فقط راستای محور زمین جاشو عوض میکنه
اگه غلط بود ببخشید:thumbsup:

Astronomy
01-13-2012, 11:02 PM
7-حمل که جابجا نمیشه! فقط راستای محور زمین جاشو عوض میکنه
فکر کنم منظورشون نقطه اول حمل بود که در اون صورت بله جابجا میشه
اگر هم منظورشون اون نبود که خوب نه جابجا نمیشه

یزدان بابازاده
01-21-2012, 12:27 PM
سلام بر دوستان پیگیر علم نخ سوزن نجوم ! پس از مدت ها غیبت یک سوال مکانیک از کتاب مکانیک کلپنر براتون می ذارم سطحشم متوسطه ولی من خیلی از سوالش خوشم اومد گفتم خوبه شما هم روش فکر کنید. " یک سفینه رای بررسی سیاره ای به جرم M و شعاع R پرتاب می شود . هنگامی که در فاصله ی 5R از سیاره است بدون حرکت معلق است بسته ای از وسایل را با سرعت v0 شلیک می کند . جرم بسته m است و به مراتب از سفینه کوچک تر است . این بسته با چه زاویه ای پرتاب شود تا در آستانه ی برخورد تماسی با سیاره قرار گیرید .
راهنمای هم خواستید می کنم.
موفق باشید.

solh
01-21-2012, 06:08 PM
سلام بر دوستان پیگیر علم نخ سوزن نجوم ! پس از مدت ها غیبت یک سوال مکانیک از کتاب مکانیک کلپنر براتون می ذارم سطحشم متوسطه ولی من خیلی از سوالش خوشم اومد گفتم خوبه شما هم روش فکر کنید. " یک سفینه رای بررسی سیاره ای به جرم M و شعاع R پرتاب می شود . هنگامی که در فاصله ی 5R از سیاره است بدون حرکت معلق است بسته ای از وسایل را با سرعت v0 شلیک می کند . جرم بسته m است و به مراتب از سفینه کوچک تر است . این بسته با چه زاویه ای پرتاب شود تا در آستانه ی برخورد تماسی با سیاره قرار گیرید .
راهنمای هم خواستید می کنم.
موفق باشید.
با توجه به این که مثال تقریبا مشابهی در کتاب هست ، زیاد سخت نیست !!:thumbsup:

انرژی اولیه رو داریم ، انرژی در هنگام اتصال رو هم داریم ، که تنها مجهول سرعت در هنگام برخورد سطحیه ، سرعت رو بدست میاریم . حالا تکانه هم پایسته است ، و تکانه هنگام برخورد رو داریم (جرم در شعاع در سرعت که به دست آمد ) ، تکانه اولیه هم تنها مجهولش زاویه پرتابه که بدست میاد .(خلاقیت رو حال کردین (خلاقیت کلپنر البته !!))

یزدان بابازاده
01-28-2012, 09:12 PM
فرض کنید یک میله یکنواخت به طول l را در صورتی که ته میله روی زمین است با زاویه ی 45 درجه نسبت به زمین خم کرده و رها می کنیم . لحظه ی برخورد میله با زمین ته میله از محل اولیه چقدر جابجا شده است؟؟!

یزدان بابازاده
01-30-2012, 07:41 PM
آقا حلال واسه این سوال نبود؟؟؟!

shokolat_g
01-30-2012, 09:19 PM
این سوال خودتون طرح کردین؟؟؟خیلی سوال جالبی!! من دارم پست میذارم ک جوابشو ندین .تا فکر کنییییییییییییییییییییییی یییییییییییییییم:dاصن این تاپیک ندیدم!!!نمیدونم کی این سوالو گذاشتین..

poorya pvp
01-31-2012, 11:51 AM
آقا حلال واسه این سوال نبود؟؟؟!

سلام.ببینید این روشو(ماله خودمه) نمیدونم درسته یا نه ولی میگم قضاوت با خودتونه :) .(با فرض اینکه سطح بدون اصطکاک باشه)

چون نیرو به صورت عمود بر سطح وارد میشه و با توجه به اینکه میتونیم فرض کنیم که کل نیرو به مرکز جرم وارد میشود میتونیم بگیم که اگر اصطکاک وجود نمیداشت،مرکز جرم به صورت عمودی پایین می آمد یعنی انتهای میله از مکان اولش جابه جا شده. و فرض کنید که اصطکاک بی نهایت باشد بنابراین انتهای میله ساکن میماند و سر میله یک دایره را به شعاع طول میله طی میکند. (این ایده ی مسئله بود روش ریاضیشو نمیتونم بنویسم :( اگه بازم متوجه نشدید،در خدمتم) با توجه به این ایده مسئله حل میشود

شرمنده قدرت بیانم خوب نیست

یزدان بابازاده
01-31-2012, 03:58 PM
سوال طرح خودمه ولی ممکنه تو کتاب های دیگه ای هم باشه نمی دونم . رو شتون هم کاملا درسته روش ریاضی اش هم ساده است دوستان بدست بیارند.

shokolat_g
02-03-2012, 12:07 PM
با یکی از دوستام حل کردیم(البته بیشتر دوستم فکر کرد)...اسکن کردیم...ولی چجوری بذارم؟؟!!!

Sunrise
02-03-2012, 12:12 PM
با یکی از دوستام حل کردیم(البته بیشتر دوستم فکر کرد)...اسکن کردیم...ولی چجوری بذارم؟؟!!!

اگر اسکن کردید و با فرمت تصویر ذخیره شده که یه جا آپش کنید و مثل یه عکس در پستتون قرار بدید

آپلود سنتر افلاک جای خوب و مطمئنی هست - لینکش بالای صفحه است ;)

هر طور شد بذارید من براتون درستش میکنم

یزدان بابازاده
02-03-2012, 12:39 PM
خب مثه اینکه سوال قبلی به یه جاهایی رسیده پس یه سواله دیگه ای می ذارم :
در روزی که ماه در بدر کامل است و در ارتفاع 90 درجه است و از قضا در سروسو نیز قرار دارد ! می خواهیم موشکی را با سرعت اولیه ی v0 مستقیما به سمت بالا پرتاب کنیم تا به ماه برسد . حداقل سرعت را بدست بیاورید ؟ از چرخش زمین و ماه نیز صرفنظر کنید.مسئله را تا جایی که ممکن است پارامتری حل کنید در نهایت کمی هم از اطلاعات آماده در کتب یا تخمین استفاده کنید.

poorya pvp
02-03-2012, 10:43 PM
خب مثه اینکه سوال قبلی به یه جاهایی رسیده پس یه سواله دیگه ای می ذارم :
در روزی که ماه در بدر کامل است و در ارتفاع 90 درجه است و از قضا در سروسو نیز قرار دارد ! می خواهیم موشکی را با سرعت اولیه ی v0 مستقیما به سمت بالا پرتاب کنیم تا به ماه برسد . حداقل سرعت را بدست بیاورید ؟ از چرخش زمین و ماه نیز صرفنظر کنید.مسئله را تا جایی که ممکن است پارامتری حل کنید در نهایت کمی هم از اطلاعات آماده در کتب یا تخمین استفاده کنید.

مگه ارتفاع 90 درجه به غیر از سرسو جای دیگه ای هم میشه؟؟؟؟؟؟؟؟؟ ;)
شتاب گرانش رو داریم. چون گفته حداقل،پس سرعت نهایی(سرعت قبل از برخورد با ماه) رو صفر در نظر میگیریم.فاصله بین زمین تا ماه رو هم داریم و چون ماه دقیقا در سرسو است و موشک را به صورت عمود پرتاب میکنیم،پس موشک روی خط راست حرکت میکند.از قوانین سینماتیک راحت حل میشه دیگه.(البته اگه اشتباه گفتم،بهم بگید لطفا :) .ممنونم)

یزدان بابازاده
02-04-2012, 04:10 PM
آره راست میگی ها!! حواسم نبود!!
از کار و انرژی راحت تره! خود دانید!

poorya pvp
02-05-2012, 01:55 PM
آره راست میگی ها!! حواسم نبود!!
از کار و انرژی راحت تره! خود دانید!
عزیزم فک کنم 1 اشتباه خیلی کوچیک دیگه هم کردید.اون قانون کار و انرژی که توی دبیرستان میخونیم،دقیقا از همون روابط سینماتیکی ای که گفتم اثبات میشه
:)

یزدان بابازاده
02-05-2012, 03:41 PM
مگه تو دبیرستان هم کار و انرژی درس می دن؟؟! به نظر من روش کار و انرژی به طور کای (انتگرالیش) هم درک بهتری درباره ی سوالات این جوری می ده و هم صورت راحت تری داره حالا شما خیلی دوست دارید از سینماتیک برید !

poorya pvp
02-06-2012, 02:52 PM
مگه تو دبیرستان هم کار و انرژی درس می دن؟؟! به نظر من روش کار و انرژی به طور کای (انتگرالیش) هم درک بهتری درباره ی سوالات این جوری می ده و هم صورت راحت تری داره حالا شما خیلی دوست دارید از سینماتیک برید !
برای من فرقی نمیکنه که از چی برم.پیش خودم فک کردم که شاید سینماتیک برای شما راحت تر باشه ولی ماشالا مغزتون نسبت به ما بازتره و راحت تر فهمیدین.
اره توی دبیرستان کار و انرژی هم درس میدن و اتفاقا از سینماتیک اثبات میشه
:)

shokolat_g
02-06-2012, 05:40 PM
دوستان....یه خواهشی دارم....یه سوال از مغناطیس بذاربد ک نجومی هم باشه....ببینیم چه جوریه!!!!!!!


مرسییییییییییییییی

poorya pvp
02-06-2012, 09:30 PM
دوستان....یه خواهشی دارم....یه سوال از مغناطیس بذاربد ک نجومی هم باشه....ببینیم چه جوریه!!!!!!!


مرسییییییییییییییی
اثبات کنید هرگاه بردار های میدان مغناطیسی ، میدان الکتریکی و سرعت در جهت بردارهای یکه دستگاه مختصات متعامدی باشند مسیر حرکت یک ذره باردار ، یک چرخزاد و در صفحه ای عمود بر بردار میدان مغناطیسی خواهد بود(اقتباس از مرحله اول پنجمین المپیاد نجوم)
:)

یزدان بابازاده
02-06-2012, 10:25 PM
برای من فرقی نمیکنه که از چی برم.پیش خودم فک کردم که شاید سینماتیک برای شما راحت تر باشه ولی ماشالا مغزتون نسبت به ما بازتره و راحت تر فهمیدین.
اره توی دبیرستان کار و انرژی هم درس میدن و اتفاقا از سینماتیک اثبات میشه
:)
دو بار گفتم که شما از هر راهی که می خواهید برید هیچ کس هم جلو تون رو نمی گیره "از هر راهی " حالا چه ذهنتون بازه چه بسته !
آهان منظورتون همون صورت بدون انتگرالیشه؟؟!

poorya pvp
02-07-2012, 12:01 AM
دو بار گفتم که شما از هر راهی که می خواهید برید هیچ کس هم جلو تون رو نمی گیره "از هر راهی " حالا چه ذهنتون بازه چه بسته !
آهان منظورتون همون صورت بدون انتگرالیشه؟؟!

حالا چرا عصبانی میشین؟؟؟؟؟
اره

یزدان بابازاده
02-07-2012, 04:00 PM
فکر کن دو بار یه چیزیو بگی بعد مجبور شی یه بار دیگه هم بگی .
صورت بدون انتگرال گیریش واقعا کاربردش کمه ، بدون انتگرال کارو انرژی به درد نمی خوره که.

یزدان بابازاده
02-23-2012, 08:18 AM
خب سلام بر دوستان همیشه پیگیر این تاپیک!!! قبل از اینکه بخوان اینجا رو به خاطر گرد و غبار زیادی ببندند یک سوال می ذارم بد نیست روش یه خورده فکر کنید:
دو نقطه داریم به مختصات فی و لاندای (120،30) و (100،45) .از این دو تا شهر دو تا دایره عظیمه رد می کنیم (از قطب به قطب) .مساحت قاچی که توسط این دو تا پوشیده شده است چقدر است؟ (درجه سختی سوال : خیلی خیلی آسون!!!)

poorya pvp
02-23-2012, 09:34 AM
نمیدونم درسته یا نه
سریع حساب کردم :
40.50936779
؟؟؟؟؟؟؟؟؟؟؟؟؟؟؟؟؟؟؟؟؟؟؟؟؟

یزدان بابازاده
02-23-2012, 11:21 PM
اصلا جواب آخر مهم نیست ، راه حل ؟؟

poorya pvp
02-24-2012, 08:28 AM
خوب 1 مثلث داریم.3 تا زاویه را به دست میاریم بعدش توی فرمول مساحت میذاریم(کتاب فاندمنتال)

یزدان بابازاده
02-24-2012, 09:13 AM
مساحت قاچو می خوایم ها

solh
02-24-2012, 10:41 AM
مساحت قاچو می خوایم ها

یه کره میکشیم ( همون دایره خودمون) ، مختصات رو مشخص میکنیم ، قاچ ها رو رسم میکنیم ، استوا رو میکشیم ، قاچ به دو مثلث مساوی تقسیم میشه ، در مثلث فرضا بالایی ، دو زاویه ی 90 درجه داریم ( محل برخورد قاچ ها با استوا ) ، و زاویه ی بالایی که برابر اختلاف لاندا هست ، مساحت رو بدست آورده (مجموع زاویه ها منهای پی کلا در شعاع کره به توان دو ، که میشه اختلاف لاندا در شعاع به توان دو )ضرب در دو میکنیم !!

یزدان بابازاده
02-24-2012, 07:59 PM
کاملا ددسته!

Mostafa
06-02-2012, 11:51 PM
خب ...

با توجه يه نزديك شدن گذر زهره و همچنين به جهت فعال سازي تاپيك هاي المپيادي كه به دليل اتمام المپياد امسال ، دارند خاك مي خورند ، يك مسئله متوسط نجوم كروي

و البته كاملآ كاربردي طرح مي كنم :

طبق برنامه رصدي آوااستار ، قراره ما اين گذر رو از محوطه كتابخانه ملي ايران رصد كنيم

تعيين كنيد كه در روز گذر خورشيد در كتابخانه ملي با چه موقعيتي نسبت به كوه دماوند طلوع ميكنه ( قابل توجه عكاسان محترم ;) )

( مختصات قله دماوند و همچنين كتابخانه ملي در بزرگراه حقاني رو هم يكي از دوستان از گوگل ارت ( ارث ! ) زحمتش رو بكشه لطفآ )

منتظريم ;)

Amirali
06-04-2012, 10:57 PM
متاسفانه طبق محاسباتی که انجام دادم خورشید در لحظه طلوع و قله دماوند حدود 8 درجه اختلاف سمت دارند
سمت طلوع خورشید: 61.6 سمت قله دماوند: 69.5
(محاسبات برای ناظر واقع در کتابخانه ملی با مختصات: 35 درجه و 45 دقیقه شمالی و 51 درجه و 26 دقیقه شرقی انجام شده است)
مختصات قله دماوند: 35 درجه و 57 دقیقه شمالی و 52 درجه و 6 دقیقه شرقی

قابل توجه کتابخونه ایا : شماها 11.6 ثانیه گذر رو بیشتر از میلادی ها میبینید اگه اونا در محوطه برج میلاد باشن
ولی اگه توی برج و در ارتفاع 400 متر بالاتر از شما باشن اونا حدود 2 دقیقه بیشتر گذر رو میبینن :دی
--------------------------------------------------------------------
پ.ن : عددهای به دست آمده ممکن است اشتباه باشند

Ehsan
06-04-2012, 11:29 PM
سوای ِ جذابیت های ِ نظری-ریاضی ِ این سوال اولا عنایت داشته باشید که پای ِ برج ِ میلاد حدودا 50 متر بالاتر از محل کتاب خونه ی ِ ملی هستش (الان از گوگل ارث دیدم) و این یعنی حدود ِ بیست و چند کیلومتر جلو افتادن ِ اونها! اما در واقع به صورت ِ عملی به خاطر ِ وجود کوه برج ِ میلادی ها زود تر می بینن! (بیشتر گذر طول می کشه)

البته این کسی رو نگران نکنه! این اختلاف دست ِ بالا یک دقیقه است!!!!

صرفا خواستم به جنبه های ِ عملی ِ این سوال اشاره کرده باشم!

پیمان اکبرنیا
06-04-2012, 11:40 PM
و البته تمام محاسبات شما در صورتی درست است که ارتفاع افق دقیقا صفر باشد که اینطور نیست و در دو محل باید اندازه گیری شود! می توانید لجظه طلوع را از طریق تلفن با برج میلادیها چک کنید هرکس زودتر خورشید را دید برنده می شود ;)

Mostafa
06-04-2012, 11:48 PM
و البته تمام محاسبات شما در صورتی درست است که ارتفاع افق دقیقا صفر باشد که اینطور نیست و در دو محل باید اندازه گیری شود! می توانید لجظه طلوع را از طریق تلفن با برج میلادیها چک کنید هرکس زودتر خورشید را دید برنده می شود ;)

اتفاقآ يك چنين قصدي هم داريم پيمان جان ;)

ضمن اينكه جايي كه ما براي رصد در نظر گرفتيم در بالاترين نقطه يك تپه است كه فكر ميكنم هم ارتفاع مينانه ارتفاع برج باشه و از سطح محوطه برج ميلاد قطعآ بالاتره

حالا حتي ميتونيم يك پروژه مشترك با بچه هاي برج ميلاد هم تعيين كنيم ! :پي

Ehsan
07-10-2012, 08:09 AM
یک مسئله ی ِ خیلی جالب به ذهنم رسیده که البته مربوط می شه به کار های ِ این روزام! :دی

وقتی شما می خواهید از یک طیف سنج استفاده کنید خیلی جالب نیست که اون رو به تلسکوپ وصل کنید و به سمت ِ خورشید بگیرید چون طیفسنج خیلی سنگینه و این کار هم ممکنه موجب ِ آسیب دیدن ِتلسکوپ بشه و هم طیف سنج!

راه ِ معقول اینه که طیف سنج یک جا ثابت باشه و نور ِ خورشید (یا ستاره ) به نحوی از یک سمت ِ همیشه ثابت (نسبت به طیف سنج) به طیفسنج برسه. خوب این کار با یک یا چند تا آینه به سادگی انجام میشه. کافیه شما یک آینه داشته باشید که همیشه نور ِ خورشید رو به سمت ِ طیف سنج منعکس کنه. فرض کنید ما آیینه به دست در مرکز ِ دستگاه ِ مختصات ایستادیم و طیف سنج درست در سمت ِ جنوب ِ ما و در ارتفاع ِ صفر هستش. معادله ی ِ حرکت ِ بردار ِ یکه ی ِ عمود بر سطح ِ آیینه رو به دست بیارید. (در واقع معادله ی ِ این که آیینه در هر لحظه باید به چه جهتی باشه)

راهنمایی:1. ناظر در نیمکره ی ِ شمالی قرار داره و عرض ِ جغرافیایش معقول هستش. 2.بردار ِ عمود بر سطح ِ آیینه همیشه نیمساز ِ زاویه ی ِ خورشید-ما-طیفسنج هستش.
3. من خودم از راه ِ برداری حلش کردم و حدس می زنم از راه ِ کروی پیرتون در بیاد تا حلش کنید! :دی



پ.ن: این مسئله خیلی خام به ذهنم اومد و ممکنه بعدا چند قسمت بهش اضافه بشه! :پی

المپیاد نجوم
08-24-2012, 07:47 PM
سلام آقا این همه راهنمایی کردین باز من نمی تونم این مسئله رو حل کنم!میشه لطفا بازم کمک کنید؟

Ehsan
08-24-2012, 08:17 PM
سلام آقا این همه راهنمایی کردین باز من نمی تونم این مسئله رو حل کنم!میشه لطفا بازم کمک کنید؟

خوشحالم بالاخره یکی تلاش کرد!

چند گام می گم واسه حلش:

1.معادله ی ِ بردار ِ یکه ای که به سمت ِ خورشید هستش رو بر حسب ِ زمان پیدا کنید
زیر گامها:
1.1. بردار ِ یکه ای در نظر بگیرید که با صفحه ی ِ افق زاویه ی ِ delta میسازه (delta همون میل ِ خورشیده)
2.1. این بردار رو با سرعت ِ زاویه ای ِ یک دور در 24 ساعت حول ِ محور ِ z دوران بدید. (از ماتریس ِ دوران حول ِ z استفاده کنید که زاویه اش wt هستش که w سرعت زاویه ای و t زمان هستش)
3.1.حالا این بردار که تابع ِ زمان هستش به اندازه ی ِ متمم ِ عرض ِ جغرافیایی دوران بدید طوری که انگار خورشید داره تو آسمون ِ واقعی می چرخه و بردار ِ عمود بر صفحه ی ِ دوران ِ خورشید به سمت ِ ستاره ی ِ قطبی خواهد بود. (از ماتریس ِ دوران استفاده کنید، این که حول ِ x باشه یا y بستگی به این داره که سمت ِ جنوب و شمال و شرق و غرب رو چه طور با این محورها منطبق کنید)

2.نیمساز ِ این بردار رو با بردار ِ یکه ای که به سمت ِ جنوب هستش رو پیدا کنید. این هم خیلی سخت نیست. دو تا بردار رو با هم جمع کنید! چون اندازه ی ِ هر دو بردار برابر هستش حاصل ِ جمع در جهت ِ نیمساز خواهد بود و بردار ِ حاصل رو نرمال کنید تا اندازه اش یک بشه (اون رو تقسیم بر اندازه اش کنید).

دیگه تقریبا راه ِ حلش رو گفتم! :دی

smhm
09-05-2012, 10:46 AM
من راه کروی مسئله را حل کردم. (البته اول از راه بردار حل می کنیم بعد بردار را تبدیل به قطبی می کنیم.):

اگر z زاویه سمت خورشید و h ارتفاع خورشید و 'h ارتفاع خورشید در نقطه x باشه. در اینصورت جهت آیینه (سمت و ارتفاع نقطه عمود بر سطح آیینه) عبارتند از:

سمت آیینه arctan [(sin z. cos h) / (1 + sin h')] ll

ارتفاع آیینه arcsin [ sin h / sqrt(2 + 2 sin h') ] ll

--------------------------
پی نوشت 1: زاویه سمت از جنوب به سمت غرب باید در نظر گرفته شود.
پی نوشت 2: نقطه x مکانی است دقیقا 90 درجه زیر شهر شما به سمت قطب جنوب.
پی نوشت 3: sin h'= cos z. cos h
پی نوشت 4: ll برای تایپک درست فرمول استفاده شده.

Amir Shayan Nejati
09-05-2012, 10:48 PM
یه سوال
دنباله دارهانوعادارای دودم هستند.یک تقریب این است که فرض کنیم یکی ازدم های دنباله دارخلاف راستای خورشیداست ودیگری درخلاف جهت سرعت فضایی دنباله داراست.اگردنباله داری درمداری باخروج ازمرکز 01/34ونیم محور(فاصله ی بین راس ها)a=1/65auحرکت بکند حساب کنیدهنگامی که درفاصله ی 1auازخورشیدقرارداردزاویه ای که دودمش باهم میسازندچقدراست؟

المپیاد نجوم
09-05-2012, 11:21 PM
با توجه به این که خروج از مرکز و نیم محور اطول رو داریم مدار مشخصه و می تونیم تتا رو پیدا کنیم و جای دنباله دار معلوم شه.با کمک این که نوع مدارو می دونیم تکانه زاویه ایشم پیدا می کنیم و باز می تونیم سرعت مماسی رو در بیاریم.با مشتق گرفتن از رابطه ی مربوط به شعاع میشه سرعت شعاعی رودر آورد چون تتا رم می دونیم.حالا سرعت کل رو داریم.شعاعم داریم و تکانه زاویه ای رم داریم.سینوس زاویه ی بین اونا رو در میاریم چون ضربمون کراسه.فک کنم 180 منهای اون زاویه ی مورد نظرمون باشه.ولی فک کنم اشتباهه اگه غلطه بهم بگید کجاش تا بیشتر فک کنم!

Amir Shayan Nejati
09-05-2012, 11:27 PM
ببین درواقع همه جای جوابت درسته به جزتعبیرهندسی که ازاون دنباله دارداری.تاسرعت مماسی رودرست رفتی ولی اگه دقت کنی ما نوک دنباله دارمون یه بردارسرعت شعاعی داریم که همون دم اولی هست ودم دیگری درخلاف جهت سرعت فضایی.بنابراین بایدزاویه بین سرعت شعاعی وسرعت فضایی روحساب کنی.(اگه توی ذهن تجسم کنیم میبینیم که بایدزاویه ی بین سرعت شعاعی وسرعت فضایی روحساب کنیم)

المپیاد نجوم
09-06-2012, 12:09 AM
ها راس میگی. من فکر کردم بین شعاع و فضایی باید باشه.درسته مر30!

celestial boy
09-06-2012, 01:00 PM
سلام.
مگه سرعت شعاعی در راستای شعاع مداری (راستای خورشید-دنباله دار) نیست؟؟خب پس اشتباه المپیاد نجوم کجا بود؟؟
سرعت فضایی در جهت حرکت حرکته؟؟

Amir Shayan Nejati
09-06-2012, 02:33 PM
سلام.
مگه سرعت شعاعی در راستای شعاع مداری (راستای خورشید-دنباله دار) نیست؟؟خب پس اشتباه المپیاد نجوم کجا بود؟؟
سرعت فضایی در جهت حرکت حرکته؟؟
سلام
قسمت اولی که گفتی رومنم تاییدکردم که درست گفته ولی خودشم توی ارسال آخرش نوشته اشتباهش چی بوده درواقع بایدزاویه ی بین بردارسرعت شعاعی وسرعت فضایی روحساب میکرده ولی زاویه ی بین شعاع وسرعت فضایی روحساب کرده اگه هنوزمشکل داری تصورش کن یااگه بازم باحرف من رونفهمیدی ازآقای شریعت زاده سوال کن من خودم فکرشمارومیکردم ولی ایشون آگاهم کرد.
اندازه سرعت و اندازه سرعت شعاعی روحساب بعد نسبتشون میشه کسینوس زاویه ای که داریم .
حالا اندازه سرعت مماسی که میشه اندازه تکانه زاویه ای تقسیم بر اندازه بردار شعاعی . از روی انرژی مدار هم سرعت فضایی.با سرعت فضایی و سرعت مماسی یه فیثاغورث، سرعت شعاعی در میاد . بعد هم اون زاویه ای که گفته روپیدامیکنیم.

المپیاد نجوم
09-06-2012, 07:04 PM
ولی ببین این راهی که من میگمم الان که فکر کردم به نظرم غلط نمیاد.
چون سرعت شعاعی در راستی شعاعه و با ضرب کراس هم منطقی به نضر میاد کاری که کردم من درست هنوز اشتباهو نفهمیدم!

Amir Shayan Nejati
09-06-2012, 07:48 PM
ولی ببین این راهی که من میگمم الان که فکر کردم به نظرم غلط نمیاد.
چون سرعت شعاعی در راستی شعاعه و با ضرب کراس هم منطقی به نضر میاد کاری که کردم من درست هنوز اشتباهو نفهمیدم!
منم اینونگفتم اشتباهه ولی خودت توپست دومت نوشتی که اول زاویه بین شعاع وسرعت فضایی روحساب کردی من اونوگفتم که نبایدزاویه بین اون دوتاپیداکنی بلکه بایدزاویه بین سرعت شعاعی وفضایی روحساب کنی.اگه بازم اشتباه داری ازآقای شریعت زاده بپرس ایشون منوآگاه کردن البته فکرکنم باضرب کراسی که میگی که سینوسشودرمیاریم درسته.

Amir Shayan Nejati
09-07-2012, 12:02 PM
حالاکه نیل آرمسترانگ که خدایش رحمت کند فوت کرده گفتم بدنیست به یادش یه سوال بزاریم:(سوال سختی نیست)
اندازه جای قدم نیل آرمسترانگ برروی سطح <<ماه>>۳۰سانتی متراست.برای این که ازروی زمین این ۳۰سانتی متری را ببینیم به تلسکوبی حداقل با چه قدرت تفکیکی نیازداریم؟(اولین کارسوق ملی المبیادنجوم سمباد)(ببخشید من حرف( ب بادونقطه زیرش )راندارم به جاش ب مینویسم)

المپیاد نجوم
09-08-2012, 07:59 PM
خوب حالا که دوستان دو تا جزوه خوب در تاپيك " نجوم كروي از ديدگاه برداري " گذاشتند من هم دو تا سوال مخصوص کسایی که علاقه به...کاری دارن میذارم.

هرکس حل کرد جوابشو بگه تا چک کنیم!

1.ماهواره ای دارای درخشندگی ذاتی با میل مداری 45 درجه و شعاع مداری 4برابر شعاع زمین توسط ناظری در عرض جغرافیایی 20 درجه در لحظه ای در نقطه

سرسو دیده می شود.چه قدر بعد از این لحظه ماهواره از دید همان ناظر غروب می کند؟


2.ماهواره ای در ارتفاع 3برابر شعاع زمین از سطح زمین در مداری دایره اب و هم صفحه با استوا به دور زمین می گردد.از دید ناظری در عرض جغرافیایی 30 درجه

شمالی برروی سطح زمین و ناظری در ارتفاع 400 کیلومتر از سطح زمین و در همان نقطه ی ناظر اول،غروب این ماهواره حداکثر چند ساعت تفاوت دارد؟

LGH
09-12-2012, 09:03 PM
حالاکه نیل آرمسترانگ که خدایش رحمت کند فوت کرده گفتم بدنیست به یادش یه سوال بزاریم:(سوال سختی نیست)
اندازه جای قدم نیل آرمسترانگ برروی سطح <<ماه>>۳۰سانتی متراست.برای این که ازروی زمین این ۳۰سانتی متری را ببینیم به تلسکوبی حداقل با چه قدرت تفکیکی نیازداریم؟(اولین کارسوق ملی المبیادنجوم سمباد)(ببخشید من حرف( ب بادونقطه زیرش )راندارم به جاش ب مینویسم)

درود!
با اجازه پیشکسوتا:دی

رد پا از زمین تحت زاویه ای دیده میشه که اینجوری بدست میاد( 2arctan(0.15/384000000 چون زاویه خیلی کوچکه پس اگه برحسب رادیان بیان شه میشه تانژانتشو تقریبا با خودش برابر در نظر گرفت پس( a=(0.3/384000000 این حداقل(؟) زاویه تفکیک تلسکوپ برای تفکیک این ردپاست البته بر حسب رادیان!

ali493
09-17-2012, 11:09 AM
فکر کنم چیزی حدود 2.5 درصد می شودبه نظر من باید طور دیگری حساب کرد آگر افق کاملا صاف باشد (موانعی مانند کوه وجود نداشته باشد چشم انسان می تواند تا 100کیلومتر آن طرف تر را ببیند پس نمی توان این گونه محیط نیمکره راحساب کرد

arashgmn
09-17-2012, 01:08 PM
درود!
با اجازه پیشکسوتا:دی

رد پا از زمین تحت زاویه ای دیده میشه که اینجوری بدست میاد( 2arctan(0.15/384000000 چون زاویه خیلی کوچکه پس اگه برحسب رادیان بیان شه میشه تانژانتشو تقریبا با خودش برابر در نظر گرفت پس( a=(0.3/384000000 این حداقل(؟) زاویه تفکیک تلسکوپ برای تفکیک این ردپاست البته بر حسب رادیان!
بهتره که شعاع زمین و ماه هم در نظر گرفته بشه چون اگه حساب کنید میبینید که واقعا تاثیر گذاره . (خطای نسبی چیزی در حدود 76 درصد خواهد بود !!!)

Amir Shayan Nejati
09-19-2012, 10:13 PM
درود!
با اجازه پیشکسوتا:دی

رد پا از زمین تحت زاویه ای دیده میشه که اینجوری بدست میاد( 2arctan(0.15/384000000 چون زاویه خیلی کوچکه پس اگه برحسب رادیان بیان شه میشه تانژانتشو تقریبا با خودش برابر در نظر گرفت پس( a=(0.3/384000000 این حداقل(؟) زاویه تفکیک تلسکوپ برای تفکیک این ردپاست البته بر حسب رادیان!
اینم جوابش که ازراه حل های دیگه خیلی راحت تره ولی اگرازراهی که که آقای گل محمدی فرمودند بریدهم جوابتون کاملادرسته...
30cmاززمین روی سطح ماه بازاویه تتادیده میشه وdهم فاصله ی ماه وزمین هست.
a=30cm
d=376000000
a/d=θ(rad
0,0002=206265*0,30/376000000=206265a/d=θ(arcsecond
جواب نهایی:0.0002

Amir Shayan Nejati
09-19-2012, 10:24 PM
یه سوال دیگه که بیشتربااین زاویه هاوتخمین آشناشیم...البته این تخمین روبایدحساب کرد!!!!!....
اندازه قطرظاهری متوسط ماه وخورشیدباهم برابر ومساوی۰٫۵درجه میباشد.باتوجه به اینکه فاصله متوسط ماه اززمین ۳۸۰هزارکیلومتراست تخمین بزنیدچندتاماه درون خورشیدقرارمیگیرد؟

celestial boy
09-20-2012, 02:46 PM
سلام.
یعنی تو این مسئله تخمینی باید فقط از همین اطلاعات استفاده کنیم؟؟یعنی فاصله زمین خورشیدو نداریم؟؟
اگه داشته باشیم مقدار تخمین میشه توان سوم نسبت فاصله زمین خورشید به زمین ماه.

Amir Shayan Nejati
09-22-2012, 04:24 PM
سلام.
یعنی تو این مسئله تخمینی باید فقط از همین اطلاعات استفاده کنیم؟؟یعنی فاصله زمین خورشیدو نداریم؟؟
اگه داشته باشیم مقدار تخمین میشه توان سوم نسبت فاصله زمین خورشید به زمین ماه.
سلام
بااین اطلاعاتی که میگم بایدحل بشه:قطرخورشید-فاصله تازمین-قطرماه-فاصله ماه تازمین.
منظورم ازحساب کردن تخمین اینه که بایدحل کنی وبعدعددگزاری وبعدم جوابوبه دست بیاری به صورت عدد.
حجم هم یادت نره اینجاکاربردداره.(ازهمین حجمه هست که مابه توان سوم نسبت فاصله زمین خورشید به زمین ماه میرسیم)(باساده کردن)
برای حل این نوع سوال هاحتمابایدحجم دوجسم روباهم مقایسه کنیم.ولی درکل جوابی که دادی درسته....
نسبت قطرخورشیدبه قطرماه میشه:۳۹۴/۷
این عددباری شعاع هاهم درسته بنابراین:vsun/vmoon=۶۰۰۰۰۰۰۰
جواب نهایی:۶۰۰۰۰۰۰۰

Amir Shayan Nejati
09-22-2012, 04:52 PM
یه سوال اخترفیزیکی:
باتوجه به اندازه گیری های انجام شده ازروی نورسنجی وعکسبرداری وطیف سنجی ستاره سماک اعزل قدرظاهری برابر۰/۹وزاویه اختلاف منظربرابر۰/۰۱۴ثانیق قوس وطول موج۱۲۶نانومتروشدت۱حاصل شده اند.شعاع این ستاره چقدراست؟

المپیاد نجوم
09-22-2012, 10:06 PM
باید با کمک اختلاف منظر فاصله رو پیدا کنی و مدول فاصله قدر مطلق رو بهت میده و قدر مطلق و درخشندگی خورشید رو هم داری!پس درخشندگی ستارتو می یابی(!!!)
بعد جسم سیاه می گیریش(یعنی اون کدری اینارو بی خیال!)و وین دمای سطحو میده بهت.
حالا تنها نداشتت شعاعه که پیداش می کنی!

soroosh
09-23-2012, 11:17 AM
شدت 1 که گفته شده دقیقا باید باهاش چی کار کرد ؟؟؟؟؟

بدون شدت حل شد یعنی غلطه ؟

celestial boy
09-23-2012, 03:30 PM
سلام.
من نفهمیدم.یعنی الان اون طول موج همون طول موج ماکزیممه که از وین استفاده کنیم؟؟اگه نیست شاید بتونیم با شدت و طول موج از تابع پلانک دما رو حساب کنبم.ولی اون چه دمایی میشه؟؟

soroosh
09-23-2012, 03:33 PM
الآن که فکر می کنم می بینم دقیقا همین طوره باید از تابع پلانک استفاده کرد

Amir Shayan Nejati
09-23-2012, 04:50 PM
سلام.
من نفهمیدم.یعنی الان اون طول موج همون طول موج ماکزیممه که از وین استفاده کنیم؟؟اگه نیست شاید بتونیم با شدت و طول موج از تابع پلانک دما رو حساب کنبم.ولی اون چه دمایی میشه؟؟
سلام
آره بایدازقانون وین وطول موج ماکزیمم استفاده کنی نیازی به شدت نیست.
اون شدتی هم که داده فکرکنم بشه ازش استفاده کردولی راهت خیلی طولانی میشه ونیازی هم بهش نیست.

Amir Shayan Nejati
09-23-2012, 04:51 PM
الآن که فکر می کنم می بینم دقیقا همین طوره باید از تابع پلانک استفاده کرد
میشه ولی نیازی نیست.
از قانون وین استفاده کن.

Amir Shayan Nejati
09-23-2012, 05:02 PM
شدت 1 که گفته شده دقیقا باید باهاش چی کار کرد ؟؟؟؟؟

بدون شدت حل شد یعنی غلطه ؟
ببین ازراه شدت میشه رابطه ی شدت رو مینویسی وباداده هات دماو...روبدست میاری. ولی طولانیه و نیازی نیست بدون این هم درسته.

المپیاد نجوم
09-23-2012, 05:14 PM
خوب سروش درست میگه دیگه قانون وین از تابع پلانک به دست می یاد(اکسترمم نسبی)
اون شدتم واسه اینه که بیشینه ی طول موج رو بهت گفته باشه و تو دمای سطحی رو در بیاری وگرنه هر دمایی رو که نمی تونیم تو رابطه ی درخشندگی جسم سیاه بذاریم!

المپیاد نجوم
09-23-2012, 05:28 PM
فکری که من کردم اینه که بیایم از مفهوم عمق اپتیکی کمک بگیریم.
ما میگیم که شدت یک رو خودمون میدیم میبینیم تو اون جسم کدر چه قدر ادامه داره و میبینیمش حالا من این جا شدت یک رو دادم و توی یک جایی لاندا رو گرفتم که شدت همون یکه در حالیکه می دونم جسم کدره و علی الاصول اگر نور من با شدت یک توش حرکت کنه باید شدتش کم شه.پس حالا که نشده یعنی توش نرفته هنوز پس دمایی که من برای اون لاندا در بیارم باید دمای سطح باشه!البته شاید اشتباه کنم اگه اشتباهه بهم بگید!
اصلا چرا این همه بپیچونیم.شدت 1 بیشینس(با همون چیزایی که از عمق اپتیکی می دونیم)و تابع پلانکم اکسترممش رو که در آوردیم منجر به رابطه ی وین شد پس با این وجود لاندا همون لاندای بیشینس و باید وین استفاده کنیم.عمق اپتیکی اینا هم به نظرم غلطه چون اصلا توجیه مناسبی نیست که اون لاندا بیشترین لاندا باشه!

Ehsan
09-23-2012, 05:58 PM
این شدت ِ یک چرا این همه گیجتون کرده؟!؟ عمق ِ اپتیکی چرا؟

ببینید، شدت تو اخترفیزیک یه چیز ِ نسبیه، یعنی شدت تو فیزیک با شدت تو اخترفیزیک فرق داره، شدت تو فیزیک انرژیی هستش که یک جسم در واحد ِ طول ِ موج تابش می کنه، توی ِ اخترفیزیک اما انرژیی هستش که از واحد ِ طول ِ موج دریافت می کنیم و چون به انرژی ِ اصلی ِ تابش شده دسترسی نداریم بنا بر این یک معیار ِ دستی برای ِ شدت درست می کنند طوری که شدت ِ بیشینه می شه یک، و بقیه ی ِ مقادیر از اون سنجیده می شه. این «یک» یعنی شدت ِ بیشینه! به این معیار هم می گن شدت ِ نسبی، نیازی به فرمول ِ پلانک نیست! :)

المپیاد نجوم
09-23-2012, 06:08 PM
همون دیگه منم آخرش گفتم عمق اپتیکی توجیه نمی کنه و ما هم به همین نتیجه رسیدیم که این شدت بیشینس و اون تابع پلانکم واسه این گفتیم که شدت بیشینه رو به قانون وین ربطشو بگیم!مگه این درست نیس؟

Ehsan
09-23-2012, 06:21 PM
حلله! :دی فقط خواستم قطعی کرده باشم :دی

در واقع این جا تابع ِ پلانک تو یه عدد ضرب شده که مقدار ِ شدت توی ِ بیشینه بشه یک! خوب دما از این جا به دست میاد، فاصله هم هست و قدر هم هست و درخشندگی به دست میاد، با درخشندگی و دما شعاع حساب می شه :)

حالا فقط یکی یه راه ِ حل ِ تر و تمیز بگذاره.

المپیاد نجوم
09-23-2012, 10:02 PM
یه سوال اساسی خیلی وقته تو ذهنم هست.
چه طوری محاسبه می کنن که مثلا فلان بارش شهابی کی دیده می شه و کی بیشینه اش هستو ...؟

Amir Shayan Nejati
09-23-2012, 11:13 PM
حلله! :دی فقط خواستم قطعی کرده باشم :دی

در واقع این جا تابع ِ پلانک تو یه عدد ضرب شده که مقدار ِ شدت توی ِ بیشینه بشه یک! خوب دما از این جا به دست میاد، فاصله هم هست و قدر هم هست و درخشندگی به دست میاد، با درخشندگی و دما شعاع حساب می شه :)

حالا فقط یکی یه راه ِ حل ِ تر و تمیز بگذاره.
من اینجوری بدست اوردم وجوابمم درست بود.
ازاختلاف منظر که میشه یک تقسیم بر فاصله فاصلش میشه۷۱.۴۳pc
m-M=5logd-5
M=0.9-5log71.43+5=-3.37
حالا ازرابطه ی قدردرخشندگی بااستفاده ازقدرمطلق خورشیدودرخشندگی اون داریم:
Msun-Mstar=2.5logLstar/Lsun
Lstar/Lsun=2228
داده ی شدت برحسب طول موج به مامیگه که بیشترین تابش درطول موج۱۲۶نانومترصورت میگیره.
λMAX=0.00898/T
T=0.00898/126nm=23000K
ازرابطه ی درخشندگی هم شعاع ستاره برحسب شعاع خورشیدبدست میاد:
Lstar/Lsun
Rstar/Rsun=2.8
جواب آخرمیشه۲٫۸البته برحسب شعاع خورشید.

Amir Shayan Nejati
09-24-2012, 05:37 PM
فرض کنیدناظران نیمکره ی جنوبی قطب شمال روقطب جنوب فرض میکنن وطول جغرافیایی رواز۹۰درجه شرقی گرینویچ میسنجند.طول وعرض جغرافیایی جدید رابدست آورید.

المپیاد نجوم
09-24-2012, 09:17 PM
یه سوال:اراتستن و شاگردش چه طوری شعاع زمین رو(خوب البته تقریبی در آوردن؟)

soroosh
09-24-2012, 09:44 PM
یه سوال:اراتستن و شاگردش چه طوری شعاع زمین رو(خوب البته تقریبی در آوردن؟)

اراتستن می دونست که مثلا در حال حاضر در یه شهری که من اسمشو نمی دونم خورشید دقیقا بالای اون شهره بعد پا شد 1000 قدم (اگه اشتباه نکنم) به سمت جنوب رفت و ارتفاع خورشید رو در همون لحظه ی خاص در اون مکان جدید اندازه گرفت

و از این راه شعاع زمین رو بدست آورد

اگه کسی دقیق ترشو می دونه لطفا بگه

المپیاد نجوم
09-24-2012, 09:52 PM
سروش اینو می دونم همون چیزی که نوشتی از این راهو میگی؟ خطاهاشو می تونی در بیاری؟

soroosh
09-24-2012, 10:00 PM
خب وقتی خورشید بالای اون شهر اولی هستش پس تو شهر دوم هم در حال عبوره
و مقداری که اخورشید ز سرسو اومده پایین همون اختلاف عرض جغدافیاییه

حالا حاصلضرب شعاع زمین در اون زاویه ( بر حسب رادیان ) باید بشه همون 1000 قدم خودش
و از این جا شعاع زمین بدست میاد

Amir Shayan Nejati
09-25-2012, 04:08 PM
فرض کنیدناظران نیمکره ی جنوبی قطب شمال روقطب جنوب فرض میکنن وطول جغرافیایی رواز۹۰درجه شرقی گرینویچ میسنجند.طول وعرض جغرافیایی جدید رابدست آورید.
یعنی واقعااین سوال اینقدرسخت بود؟
حل کنیددیگه بدست اوردنش یه دقیقه هم زمان نمیبره.

Amir Shayan Nejati
10-03-2012, 05:53 PM
‫دوستان من این سوال روکمی اشکال دارم اگه میشه کمکم کنید...لطفادقیق بنویسید(کمی نفهمم)
یکی از سیارک های بین مر یخ و مشتری در مداری بیضوی با نیم محور بز رگ ‪ a = ۳au‬و خر وج از مرکز۰٫۵ = ‪ e‬حرکت‬
‫می کند. در اثر اغتشاشات ناشی از حرکت مشتری خر وج از مرکز مدار سیارک به طور ناگهانی زیاد می شود در حالی که انر ژی‬
‫مدارش ثابت می ماند. حساب کنید حداقل خر وج از مرکز مدار جدید چه قدر باشد تا امکان برخورد سیارک با زمین وجود داشته باشد؟‬

shariatzadeh
10-03-2012, 06:02 PM
‫دوستان من این سوال روکمی اشکال دارم اگه میشه کمکم کنید...لطفادقیق بنویسید(کمی نفهمم)
یکی از سیارک های بین مر یخ و مشتری در مداری بیضوی با نیم محور بز رگ ‪ a = ۳au‬و خر وج از مرکز۰٫۵ = ‪ e‬حرکت‬
‫می کند. در اثر اغتشاشات ناشی از حرکت مشتری خر وج از مرکز مدار سیارک به طور ناگهانی زیاد می شود در حالی که انر ژی‬
‫مدارش ثابت می ماند. حساب کنید حداقل خر وج از مرکز مدار جدید چه قدر باشد تا امکان برخورد سیارک با زمین وجود داشته باشد؟‬


برای حل سوال دو تا شرط داریم ، که هر کدوم یک نتیجه رو حاصل میکنه:
1- انرژی مدار ثابت هست ، در نتیجه : نیم قطر اطول مدار تغییر نمیکنه و همون 3 واحد نجومی میمونه.
2- حداقل خروج از مرکزی رو می خواهیم که سیارک به زمین برخورد کنه . در نتیجه: شعاع حضیض باید 1 واحد نجومی باشه .
پس خروج از مرکز 0.66 میشه .

Amir Shayan Nejati
10-03-2012, 06:34 PM
برای حل سوال دو تا شرط داریم ، که هر کدوم یک نتیجه رو حاصل میکنه:
1- انرژی مدار ثابت هست ، در نتیجه : نیم قطر اطول مدار تغییر نمیکنه و همون 3 واحد نجومی میمونه.
2- حداقل خروج از مرکزی رو می خواهیم که سیارک به زمین برخورد کنه . در نتیجه: شعاع حضیض باید 1 واحد نجومی باشه .
پس خروج از مرکز 0.66 میشه .
حالاکه نگاه میکنم مشکلم حل شد....مرسی

erfan bayat
10-04-2012, 09:07 PM
شعاع مشتری سالانه به میزان 0/66 میلیمتر کاهش میابد. به واسطه این انقباض مقداری انرژی ازاد میشود.اگر ضریب بازتاب مشتری 0/58 میزان انرژی تولید شده ناشی از انقباض گرانشی مشتری را با انرژی دریافتی خورشید مقایسه کنید.
با صرف نظر از نور باز تابی خورشید در سطح متری و با فرض این که دوره تناوب وضی مشتری کوچک است,حداقل و حداکثر قدر ان را از دید ناظر زمینی به دست اورید.(مدار زمین و مشتری دایره ای و هم صفحه بگیرید)طول موجی که مشتری در ان بیشترین تابش را دارد به دست اورید.شعاع مداری مشتری 7.78*11^10 متر -شعاع مشتری در استوا 7.15*7^10 متر-جرم مشتری 1.9 *27^10 کیلو گرم
------------------------
رو سوالا فکر کنید و عدد اخرو درارید حتمااااااااااااااا
من جواب هر سوالو که میگم میزارم

المپیاد نجوم
10-04-2012, 09:54 PM
راه حلمو میگم اگه درست بود بگید که بنویسم.
برای قسمت انقباض خودگرانشی البته من باضریب یک در نظر گرفتم.بعد پتانسیل خودگرانشی رو ازش مشتق گرفتم تا کوچک شدن سالانه وارد شه و منفی اونو برابر درخشندگی گذاشتم.
شار دریافتی از خورشید در یک منهای آلبدو در سطح رو هم در سال در آوردم.
بعد برای فاصله هم توی مقابله و مقارنه در نظر گرفتم و شار رو در آوردم و توی رابطه ی قدر گذاشتم واختلاف قدر رو پیدا کردم.البته دونه دونشم با خورشید میشه نوشت و هر کدومو در آورد.
آخرشم که وین نوشتم!

erfan bayat
10-04-2012, 10:03 PM
راه حلمو میگم اگه درست بود بگید که بنویسم.
برای قسمت انقباض خودگرانشی البته من باضریب یک در نظر گرفتم.بعد پتانسیل خودگرانشی رو ازش مشتق گرفتم تا کوچک شدن سالانه وارد شه و منفی اونو برابر درخشندگی گذاشتم.
شار دریافتی از خورشید در یک منهای آلبدو در سطح رو هم در سال در آوردم.
بعد برای فاصله هم توی مقابله و مقارنه در نظر گرفتم و شار رو در آوردم و توی رابطه ی قدر گذاشتم واختلاف قدر رو پیدا کردم.البته دونه دونشم با خورشید میشه نوشت و هر کدومو در آورد.
آخرشم که وین نوشتم!
خوبه فقط عدداشم در بیار

erfan bayat
10-05-2012, 08:09 AM
شعاع مشتری سالانه به میزان 0/66 میلیمتر کاهش میابد. به واسطه این انقباض مقداری انرژی ازاد میشود.اگر ضریب بازتاب مشتری 0/58 میزان انرژی تولید شده ناشی از انقباض گرانشی مشتری را با انرژی دریافتی خورشید مقایسه کنید.
با صرف نظر از نور باز تابی خورشید در سطح متری و با فرض این که دوره تناوب وضی مشتری کوچک است,حداقل و حداکثر قدر ان را از دید ناظر زمینی به دست اورید.(مدار زمین و مشتری دایره ای و هم صفحه بگیرید)طول موجی که مشتری در ان بیشترین تابش را دارد به دست اورید.شعاع مداری مشتری 7.78*11^10 متر -شعاع مشتری در استوا 7.15*7^10 متر-جرم مشتری 1.9 *27^10 کیلو گرم
------------------------
رو سوالا فکر کنید و عدد اخرو درارید حتمااااااااااااااا
من جواب هر سوالو که میگم میزارم
4912
راه حل قسمت اول که مشخصه فقط میبینید که میتونیم ضرب دوتا r ها رو برابر بزاریم چون عملا اختلاف کمی دارند
برای قسمت بعدی بیشترین و کمترین شار نوشتم فقط باید با قدر خورشید مقایسه کنید و قدر ها رو بدست بیارید
برای قسمت اخر هم باید دمای تعادل مشتری دراریم مطابق رابطه بالا. فقط حواستون باشه که گفته شده مشتری سریع میچرخه پس دما سریع در سطح منتقل و همدما میشه پس مساحت باید4پی2^r
باشه.اگر مشتری سریع نمیچرخید(دوره تناوب وضعی زیادی داشت)مساحت2پی2^r خواهد بود.پس از حساب دما با استفاده از قانون وین طول موج ماکزیمم به دست میاد
4913

Moris75
10-05-2012, 11:21 AM
یك سوال :

ثابت کنید فاصله ی سیاره از خورشید برحسب بی هنجاری خروج از مرکزی اش به شکل زیر است:
r=a(1-ecos(E

Amir Shayan Nejati
10-05-2012, 11:53 AM
یك سوال :

ثابت کنید فاصله ی سیاره از خورشید برحسب بی هنجاری خروج از مرکزی اش به شکل زیر است:
r=a(1-ecos(E
آنومالی خروج از مرکزی(Eccentric Anomaly)زاویه ای کمکی است که به وسیله دایره (http://www.wikiastro.ir/index.php/%D8%AF%D8%A7%DB%8C%D8%B1%D9%87) کمکی در بیضی (http://www.wikiastro.ir/index.php/%D8%A8%DB%8C%D8%B6%DB%8C) تعریف می شود. دایره کمکی ، دایره ای است که بر بیضی مماس بوده و شعاع آن برابر با نیم قطر اطول مدار بیضی است .
به شکل زیر توجه کنید :
http://www.wikiastro.ir/images/d/dc/Eccentric_Anomaly1.png (http://www.wikiastro.ir/index.php/%D9%BE%D8%B1%D9%88%D9%86%D8%AF%D9%87:Eccentric_Ano maly1.png)

در شکل بالا F کانون بیضی ،O مرکز بیضی و θ آنومالی واقعی بیضی است .
همانطور که در شکل بالا نیز نشان داده شده است ، برای محاسبه آنومالی خروج از مرکزی از مکان جسم در مدار خطی به محور اصلی عمود می کنیم(خط SV) و سپس خط را در خلاف جهت ادامه می دهیم تا دایره کمکی را در نقطه Q قطع کند . حال زاویه ای که خط OQ با محور اصلی می سازد را آنومالی خروج از مرکزی می نامیم و آن را با (E) نشان می دهیم . حال با توجه به شکل می توانیم روابط زیر را نتیجه بگیریم :
(1) http://www.wikiastro.ir/images/d/d8/Eccentric_Anomaly2.png (http://www.wikiastro.ir/index.php/%D9%BE%D8%B1%D9%88%D9%86%D8%AF%D9%87:Eccentric_Ano maly2.png)
و درنهایت :
(2)http://www.wikiastro.ir/images/5/55/Eccentric_Anomaly3.png (http://www.wikiastro.ir/index.php/%D9%BE%D8%B1%D9%88%D9%86%D8%AF%D9%87:Eccentric_Ano maly3.png)
می توانیم با استفاده از معادله بالا نتایج زیر را بگیریم :
(3) http://www.wikiastro.ir/images/8/80/Eccentric_Anomaly4.png (http://www.wikiastro.ir/index.php/%D9%BE%D8%B1%D9%88%D9%86%D8%AF%D9%87:Eccentric_Ano maly4.png)
حال با توجه به اتحاد زیر
(4)http://www.wikiastro.ir/images/8/89/Eccentric_Anomaly5.png (http://www.wikiastro.ir/index.php/%D9%BE%D8%B1%D9%88%D9%86%D8%AF%D9%87:Eccentric_Ano maly5.png)
و معادلات (3) نتیجه زیر به دست می آید:
(5)http://www.wikiastro.ir/images/e/e1/Eccentric_Anomaly6.png (http://www.wikiastro.ir/index.php/%D9%BE%D8%B1%D9%88%D9%86%D8%AF%D9%87:Eccentric_Ano maly6.png)
حال معادله (2) را به صورت زیر بازنویسی می کنیم :
(6) http://www.wikiastro.ir/images/d/d1/Eccentric_Anomaly7.png (http://www.wikiastro.ir/index.php/%D9%BE%D8%B1%D9%88%D9%86%D8%AF%D9%87:Eccentric_Ano maly7.png)
و با توجه به معادله قطبی بیضی داریم :
(7) http://www.wikiastro.ir/images/b/b4/Eccentric_Anomaly8.png (http://www.wikiastro.ir/index.php/%D9%BE%D8%B1%D9%88%D9%86%D8%AF%D9%87:Eccentric_Ano maly8.png)

Ehsan
10-05-2012, 12:25 PM
بچه ها سعی کنید مسائلی رو که اینجا طرح می کنید ایده دار و قشنگ باشن! دیگه سوالات ِ کلیشه ای همه جا پیدا می شن و سوالات ِ خرکاری هم چندان جذابیتی ندارن!

مثلا جهانی ِ 2010 چین یه سوال ِ ایده دار و قشنگ داشت که در عین ِ سادگی و محاسبات ِ کم، خیلی قشنگ و مفهومی بود:

یک فضا نورد می خواد دور ِ یک سیارک رو در عرض ِ یک ساعت با پیاده روی طی کنه، شعاع ِ سیارک 1 کیلومتر و جرم ِ سیارک 4 ضرب در ده به توان 13 کیلوگرم هستش و شکلش هم کرویه. آیا فضا نورد می تونه؟ جواب رو فقط با بله یا خیر و همراه با محاسبات ِ لازم ذکر کنید. (عدد هاش دقیقا این نبود و من از خودم در آوردم! :دی ولی خدا وکیلی خیــــــــــــــــلی سوال ِ قشنگیه!)

Moris75
10-05-2012, 03:16 PM
یک سوال ساده و جالب:وقتی من با زاویه ی خاصی به کنج سقف نگاه می کنم زاویه ی هر کدام از یال های دیوار با هم را بر حسب زاویه ها و طول بر دار چشم من بدست آورید.(اگر متوجه سوال نشدید یا توضیح بیشتر لازم است بگید تا توضیح بدم)4915

المپیاد نجوم
10-05-2012, 04:14 PM
توی سوال آقای ابراهیمیان من فکر کردم که اگه بیایم نیروی گرانشی رو با جرم در شتاب که همون مشتق سرعت نسبت به زمان میشه برابر بذاریم و انتگرال بگیریم و سرعت رو پیدا کنیم که من در آوردم 9.6 متر بر ثانیه.بعد سرعت مورد نیاز برای دور زدن رو هم پیدا کنیم ببینیم که اگه کمتر از 9.6 متر بر پانیه بود می تونه.فقط یه چیزی منظورش از دور زدن اینه که از قطب بیاد پایین و کل کره رو طی کنه یا دور استو منظورشه؟
سوال بعدیشم نتونستم صورتشو متوجه شم اگه میشه بیشتر توضیح بدید!
ممنون!

Moris75
10-05-2012, 05:03 PM
راهنمایی:صفحه ی عمود بردار چشم رو پیداکن و در ضمن به شکل هم توجه کن

Ehsan
10-05-2012, 06:06 PM
بچه ها سعی کنید مسائلی رو که اینجا طرح می کنید ایده دار و قشنگ باشن! دیگه سوالات ِ کلیشه ای همه جا پیدا می شن و سوالات ِ خرکاری هم چندان جذابیتی ندارن!

مثلا جهانی ِ 2010 چین یه سوال ِ ایده دار و قشنگ داشت که در عین ِ سادگی و محاسبات ِ کم، خیلی قشنگ و مفهومی بود:

یک فضا نورد می خواد دور ِ یک سیارک رو در عرض ِ یک ساعت با پیاده روی طی کنه، شعاع ِ سیارک 1 کیلومتر و جرم ِ سیارک 4 ضرب در ده به توان 13 کیلوگرم هستش و شکلش هم کرویه. آیا فضا نورد می تونه؟ جواب رو فقط با بله یا خیر و همراه با محاسبات ِ لازم ذکر کنید. (عدد هاش دقیقا این نبود و من از خودم در آوردم! :دی ولی خدا وکیلی خیــــــــــــــــلی سوال ِ قشنگیه!)


توی سوال آقای ابراهیمیان من فکر کردم که اگه بیایم نیروی گرانشی رو با جرم در شتاب که همون مشتق سرعت نسبت به زمان میشه برابر بذاریم و انتگرال بگیریم و سرعت رو پیدا کنیم که من در آوردم 9.6 متر بر ثانیه.بعد سرعت مورد نیاز برای دور زدن رو هم پیدا کنیم ببینیم که اگه کمتر از 9.6 متر بر پانیه بود می تونه.فقط یه چیزی منظورش از دور زدن اینه که از قطب بیاد پایین و کل کره رو طی کنه یا دور استو منظورشه؟
سوال بعدیشم نتونستم صورتشو متوجه شم اگه میشه بیشتر توضیح بدید!
ممنون!
:o
اصلا نیازی به انتگرال گیری نیست! کلا دو تا فرمول می خواید! یکیش تقسیم ِ طول به زمان هستش که سرعت ِ مورد ِ نیاز برای ِ دور زدن ِ سیارک رو به دست می ده! اون یکیش رو نمی گم چون تابلو می شه جوابش! :)) اما همین قدر بگم که فرمولش معروف و ساده است!

راهنمایی: شخص می خواد پیاده یک دور ِ کامل دور ِ سیارک بزنه اون هم در کمتر از یک ساعت! سوال اینه که می تونه؟

سرعتش رو هم زیاد پیدا کردید! این قدر نیست :)

Amir Shayan Nejati
10-05-2012, 06:17 PM
توی سوال آقای ابراهیمیان من فکر کردم که اگه بیایم نیروی گرانشی رو با جرم در شتاب که همون مشتق سرعت نسبت به زمان میشه برابر بذاریم و انتگرال بگیریم و سرعت رو پیدا کنیم که من در آوردم 9.6 متر بر ثانیه.بعد سرعت مورد نیاز برای دور زدن رو هم پیدا کنیم ببینیم که اگه کمتر از 9.6 متر بر پانیه بود می تونه.فقط یه چیزی منظورش از دور زدن اینه که از قطب بیاد پایین و کل کره رو طی کنه یا دور استو منظورشه؟
سوال بعدیشم نتونستم صورتشو متوجه شم اگه میشه بیشتر توضیح بدید!
ممنون!
برای سوال برداری صفحه عمودبربردارچشم روپیداکن وبعدبردارهایی که داریم روواردش کن بعد یه دستگاه کارتزین قراربده وزاویه هاروبدست بیار.

erfan bayat
10-05-2012, 07:36 PM
از دوستان خواهش میکنم پست الکی نزارن و پستاشون مفید باشه وگرنه پاک خواهد شد
بعد سوالایی بزارین که جوابشو بلد نیستید.اینجوری وقتتون تلف میشه .این بخش متاسفانه هنوز خیلی فعال نشده که کسی سریع جواب سوالارو بده
بعد در مورد سوال اقای ابراهیمیان
چرا یه ذره ساده فکر نمیکنید؟!!

المپیاد نجوم
10-06-2012, 08:23 PM
بچه ها بیاین این سوالو باهم حل کنیم.
اورانیوم طبیعی در پوسته ی زمین به صورت ترکیبی از اورانیوم(238)با فراوانی نسبی 99.28درصد و اورانیوم(235)با فراوانی نسبی 0.72درصد یافت می شود.
اگر فرض کنیم که در ابتدای تشکیل منظومه ی خورشیدی فراوانی این دو ایزوتوپ برابر بوده است،سن منظومه ی خورشیدی را با استفاده از قانون فروپاشی رادیواکتیو حساب کنید.نیمه عمر عناصر رادیواکتیو زمانی است که نیمی از مقدار اولیه موجود از آن فروپاشی کرده باشد!
نیمه عمر اورانیوم :7038*10^8 سال
نیمه عمر اورانیوم:4468*10^9 سال

soroosh
10-06-2012, 11:09 PM
اگه نیم عمر ها رو این طوری باشند :7038 ضربدر 8 به توان 10 و اون یکی 4468 ضربدر 9 به توان 10

اون وقت جواب من می شه حدود 10 به توان 14 سال
که به نظرم خیلی زیاده !!!

المپیاد نجوم
10-06-2012, 11:30 PM
راه حلتم بگو دیگه!

shariatzadeh
10-07-2012, 09:34 PM
ببینید این که چی درسته ، چی غلطه ، چی مفیده ، چی به درد می خوره و چی های دیگر رو یکی میگه که بفهمه اون چیز چیه !!!

اینجا مدیر داره ، چند نفر هستن که تجربه دارن . این درست نیست که هنوز از در نیمدی تو به دکوراسیون گیر بدی . برای تغییراتش برنامه ریزی کنی .

دوستان چنان درباره فواید المپیاد حرف میزنن ، هر کی نفهمه فکر میکنه طرف 10 تا مدرسه مختلف درس میده ، تمام پیچ و خم راه رو دیده . (یه نگاه به تاپیک فواید و مضرات المپیاد بندازید)

اگر یه درس جدید یاد می گیری دلیل نداره هی بیای اینجا پست بزاری . ما اینجا برای مدیریت تالارها برنامه ریزی می کنیم . که چه مطلبی کی و چه طور گفته بشه .

من به همه ی دوستانی که تازه وارد می شن اعلام می کنم ، اینجا جایی نیست که استعداد هاتون رو شکوفا کنید و بیاید درس بدید ، نکته آموزشی بگید .
اینجا (فروم) برای اینه که بحث کنید و رفع اشکال کنید ، با هم اشکالاتتون رو بررسی کنید . نه که اشکالات مرحله اول دوره قبل رو بررسی کنید .( اول به فکر پیشرفت خودتون باشید بعد به فکر پیشرفت تاپیک کیهانشناسی .)
ما هم وظیفه داریم نکات آموزشی بگیم و وقتی بحث یا سوالی رو نتونستید حل کنید راهنمایی کنیم . چون بیشتر شما کسایی هستند که عمق المپیاد رو ندیدن و این حرفی که می زنم برای پیشرفت خودتون و فروم هست .

اگر کسی چیزی گفت که شما خوشتون نیومد هم قرار نیست بزنید نابودش کنید . حالا طرف منظوری نداشته . اینجا محیط مجازیه و نمیشه احساسات رو منتقل کرد ، برای همین خیلی ناراحت نشید . نیازی نیست با کلمات هیولا و دراکولا و ... خطابش کنید .

این حرفا چند روزی بود که تو دلم مونده بود ، گفتم بگم که ممکنه کار به جاهای باریک بکشه . این نکته رو هم اضافه می کنم که از این چیزی که نوشتم ناراحت نشید ، من توی رسوندن منظور تبهر خاصی دارم و همیشه از مثال های عددی استفاده میکنم !!!

soroosh
10-07-2012, 10:16 PM
راهم که مشخصا از فرمول نیمه عمر هستش که اثباتش هم آسونه البته نمی تونم تایپش کنم اگر خواستید عکس می گیرم میذارم
توی کتاب زیلیک و اسمیت هست که البته باز هم نمیدونم کدام فصل

از توی فرمول لگاریتم فراوانی ها بدست می آید ضربدر اختلاف معکوس های نیمه عمرها

Dark Relativity
10-09-2012, 05:46 PM
با حل این سوالات میتوان درک بیشتری از حس اب دیفرانسیل بدست آورد
1-ضریب بازتاب زهره ودرخشندگی خورشید را بترتیب L , a فاصله زهره تا خورشید d فرض کنید کم دما ترین نقطه بین این دو جرم را بیابید؟
2-موجودی بین این دو جرم قرار دارد نقطه ای را پیداکنید که درآن جاذبه وارد بر موجود min باشد؟

Moris75
10-12-2012, 06:15 PM
حالا که تایپیک ما بسته شده و نمتونم توش پست بدم(از مسولین خواهش مکنم رسیدگی کنید این حرکت استبدادی آخه چرا من که با انتقالش موافقم )یه سوال میدم حال کنید تا تعریفتون از حال چی باشه::)
یک کارت با سرعت ثابت v روی یک میز رها میشود. کارت دوم که کوچکتر است در همان جهت نسبت به دکارت اول رها میشودو کرت سوم در همان جهت و با همان سرعت نسبت به کارت دوم رها میشود و این کار همین طور تا کارت n ام ادامه می یابد.الف)سرعت کارت n ام را در مختصات زیر بیابید. ب)وقتی n به سمت بی نهایت میل میکند این سرعت چه مقداری خواهد یافت؟:stupido:

المپیاد نجوم
10-15-2012, 09:30 PM
تو سوال آقای ابراهیمیان حق استفاده از تقریب رو برای قسمت پیاده روی داریم یا باید دقیق پیدا کنیم؟

erfan bayat
10-15-2012, 10:31 PM
تو سوال آقای ابراهیمیان حق استفاده از تقریب رو برای قسمت پیاده روی داریم یا باید دقیق پیدا کنیم؟
سوالو فقط لازمه ساده ببینی
راهنمایی:سرعت شخص باید یه حدی داشته باشه

المپیاد نجوم
10-16-2012, 06:57 PM
تا روی سوال قبلی هممون فکر کنیم من یه سوال دیگه که به نظرم خوب اومد رو میگم رو اینم فکر کنید.
فضاپیمایی خورشیدی را در نظر بگیرید که داراس صفحه یا بادبان خورشیدی دایره ای شکلی به شعاع Rاست.جنس آن آینه ی کامل است.در ابتدای قرارگیری فضاپیما در بالای جو در مدار زمین،صفحه ی بادبان کاملا در جهت عمود بر خط واصل خورشید _فضاپیما قرار داشت.بعد از آن زاویه ی بین خط عمود بر صفحه و بردار مکان فضاپیما (TETHA)شروع به تغییر کرد که تتا=امگا در زمان و در آن امگا همان سرعت زاویه ای بسیار کوچک دوران صفحه است.سرعت متوسط دور شدن فضاپیما از خورشید بعد از باز شدن صفحات و دوران آنها را در tپانیه ی کوتاه اول حرکت بیابید!
r را می توانید ثابت فرض کنید چون امگا خیلی کوچک است!

المپیاد نجوم
10-16-2012, 08:43 PM
به نظرم فضانورد می تونه چون سرعتی که احتیاج داره تا بتونه تویک ساعت دور بزنه میشه:1.745متر بر ثانیه و سرعت فرار از سیاره هم میشه:2.3 متر بر ثانیه که یعنی فضانورد به سرعت فرار نمیرسه و دور می زنه!
این فکر هم اشتباهه؟

Moris75
10-17-2012, 04:32 PM
تا روی سوال قبلی هممون فکر کنیم من یه سوال دیگه که به نظرم خوب اومد رو میگم رو اینم فکر کنید.
فضاپیمایی خورشیدی را در نظر بگیرید که داراس صفحه یا بادبان خورشیدی دایره ای شکلی به شعاع Rاست.جنس آن آینه ی کامل است.در ابتدای قرارگیری فضاپیما در بالای جو در مدار زمین،صفحه ی بادبان کاملا در جهت عمود بر خط واصل خورشید _فضاپیما قرار داشت.بعد از آن زاویه ی بین خط عمود بر صفحه و بردار مکان فضاپیما (TETHA)شروع به تغییر کرد که تتا=امگا در زمان و در آن امگا همان سرعت زاویه ای بسیار کوچک دوران صفحه است.سرعت متوسط دور شدن فضاپیما از خورشید بعد از باز شدن صفحات و دوران آنها را در tپانیه ی کوتاه اول حرکت بیابید!
r را می توانید ثابت فرض کنید چون امگا خیلی کوچک است!
خیلی ممنون که سوال گذاشتید اما اینجوری که سوال بقیه دیگه حل نمیشه آسیاب به نوبت دو تایی سوال تو صف بودن مثلا سوال کارت هایی که من قرار دادم.

Ehsan
10-17-2012, 04:51 PM
به نظرم فضانورد می تونه چون سرعتی که احتیاج داره تا بتونه تویک ساعت دور بزنه میشه:1.745متر بر ثانیه و سرعت فرار از سیاره هم میشه:2.3 متر بر ثانیه که یعنی فضانورد به سرعت فرار نمیرسه و دور می زنه!
این فکر هم اشتباهه؟

خــــــــــــیلی نزدیک ِ جواب شدید! منتهی سرعت ِ حدی سرعت ِ فرار نیست :) دقت کنید فضا نورد می خواد پیاده روی کنه! :)

المپیاد نجوم
10-17-2012, 09:39 PM
لطفا جوابشو بگید من برای پیاده روی حد سرعتی به ذهنم نمی رسه!

celestial boy
10-17-2012, 10:13 PM
سلام...
چون فرمودید ساده بینی احتیاج داره من بسیار ساده دیدم و سرعتی که المپیاد نجوم به دست آورده رو تبدیل به شتاب مرکز گرا (V^2/r) کردم.بعد شتاب گرانشی سیارک رو هم حساب کردم.دومی از اولی کمتر بود پس فضانورد نمیتونه.فقط:من همون رابطه سرعت فرار رو استفاده نکردم؟؟؟!!!!!!!!!!

المپیاد نجوم
10-17-2012, 10:20 PM
یه چیزی
سرعت رو چی کار کردی؟ نفهمیدم میشه راهتو توضیح بدی؟

erfan bayat
10-18-2012, 06:24 AM
یه چیزی
سرعت رو چی کار کردی؟ نفهمیدم میشه راهتو توضیح بدی؟
ببین مگه قرار نیست پیاده بره؟
پس نباید از سطح سیاره بلند شه.پس سرعت فرار اون حده نیست. سرعت ماهواره شدن مهمه.1/2^(gm/r)
حالا یه سرعتم با استفاده از شعاع سیاره و اون یه ساعته به دست میاد که باید با هم مقایسه شه

celestial boy
10-18-2012, 07:43 AM
یه سوال.الان من چیکار کردم؟؟فکر کنم منم همین کارو کردم.فقط تو رابطم کافی بود r ها رو از دو طرف بزنم.(GM/r^2>V^2/r) یعنی باید شتاب گرانشی از شتاب ایجاد شده بیشتر باشه یا اینطوری (GM/r>V^2) که بازم همون میشه.فضا نورد نمیتونه.
«حالا که نمیتونه یعنی چه اتفاقی واسش میفته؟»

المپیاد نجوم
10-18-2012, 08:38 PM
چرا شتاب هارو مقایسه می کنی؟مگه هدفمون سرعت ها نیست؟
راس میگه چه بلایی سرش میاد؟ماهواره میشه؟؟؟؟؟:d

arashgmn
10-18-2012, 08:58 PM
فضانورد میخواد که پیاده روی کنه . پس سرعت متوسطش اگه قرار باشه این کارو بکنه میشه :

V=2*pi*r/t


که t همون یک ساعته و r هم شعاع سیارکه. که عددش میشه 1.75 متر بر ثانیه .

اما اگه سرعت یه ماهوراره ای که توی ارتفاع صفر از سطح سیاره در گردشه رو حساب کنید ، عددش میشه 1.63 متر بر ثانیه . یعنی ماهواره ای که توی یه مدار دایره ای تو ارتفاع صفر در حرکته سرعتش از فضانورده کمتره .بنابراین قطعا فضانورد نمی تونه پیاده روی کنه .

پس فضانورده اگر که بخواد با همون سرعت 1.75 متر بر ثانیه حرکت کنه ، بعد از یه مدتی از سطح سیارک کنده میشه و تبدیل به یه ماهواره میشه با یه مدار بیضی (دقت کنید که سرعت فرار شکسته نشده ، پس مدار بسته می ماند.)

celestial boy
01-01-2013, 03:46 PM
سلام علیکم.
یه سوال از سوالای مرحله 2 (که نمی دونم کجاش نجومیه؟!!!)
مرحله 2 اردیبهشت 88 سوال 4.گفته جابجایی فریز ها چقدره؟سوالش طولانیه و تصویر داره نمی تونم بذارمش.
از معلمای فیزیکم پرسیدم درست و حسابی جواب ندادن.خودم میگم تو سرعت نور که اختلاف ایجاد نمیشه.مسیر ها هم یکی هستن.پس تداخل واسه چی؟؟؟

shariatzadeh
01-01-2013, 04:15 PM
سلام علیکم.
یه سوال از سوالای مرحله 2 (که نمی دونم کجاش نجومیه؟!!!)
مرحله 2 اردیبهشت 88 سوال 4.گفته جابجایی فریز ها چقدره؟سوالش طولانیه و تصویر داره نمی تونم بذارمش.
از معلمای فیزیکم پرسیدم درست و حسابی جواب ندادن.خودم میگم تو سرعت نور که اختلاف ایجاد نمیشه.مسیر ها هم یکی هستن.پس تداخل واسه چی؟؟؟

علیک سلام
من سوال رو اینجا قرار می دم .سوال 4 مرحله دوم پنجمین المپیاد نجوم :

http://up.avastarco.com/images/3vzhyv0615tvvpwn9i1.png (http://up.avastarco.com/)

ببینید توی صورت سوال گفته از اثرات نسبیتی صرفه نظر کنید . شما باید فرض کنید وقتی نور هم جهت با آب حرکت میکنه ، سرعتش با آب جمع میشه و وقتی خلاف جهت آب حرکت میکنه سرعت آب از سرعت نور کم میشه . در واقع اون موقعی که فیزو این آزمایش رو انجام داد مردم یه همچین فکری می کردن و اون با انجام آزمایش نشان داد که این فرض غلطیه .
حالا چون دو پرتو نور سرعت یکسانی ندارند با یک اختلاف زمانی به تلسکوپ می رسند ، منظور از جابه جایی فریز ، تعداد طول موجی هست که توی این مدت زمان موج طی میکنه .
این سوال رو میتونید توی تمرینات کتاب کلپنر ببینید و توی خیلی از کتاب های نسبیت هم هست .

celestial boy
01-02-2013, 02:53 PM
سلام.
آها پس این به صورت واقعی و عملی ممکن نیست.
یه سوال خارج از بحث.اگه به جای نور ،صوت یا موج دیگه ای قرار بدیم این قضیه درسته؟؟یعنی تداخل داریم؟؟چی باعث تغییر سرعت میشه؟؟
تشکر...

المپیاد نجوم
01-02-2013, 07:00 PM
سلام.منم چند تا سوال دارم.
1.توی سوالات آقای عطا مرادی یک سوال بود که می گفت در یک ستاره که 70 درصد آن هلیوم است و هسته 0.1 شعاع را تشکیل می دهد و دمایش میلیون درجه کلوین است و چگالی ثابت و جرم و شعاع هم مثل خورشیدند.حالا ما باید درصد عناصر سنگین را حساب می کردیم.
من با کمک فرض ثابت بودن چگالی جرم در آن لایه را در آوردم و تعادل هیدرواستاتیک نوشتم و با فرض گاز تماما ایده آل با ساده سازی فشار مرکزی را در آوردم و بعد هم میو را در آنجا در آوردم و با دانستن این که جمع x,zسی درصد است و این که 0.5z,2xهم برآیندشان یک میوام هست مساله رو حل کردم ولی جواب آخر را شک دارم.میشه بچه های دیگه هم جواب آخرشونو بگن چک کنیم؟
2.این سوالاو هنوز به اندازه ی کافی روش فکر نکردم شاید زیاد سخت نباشه ولی گفتم با هم فکر کنیم.
فضاپیمایی خورشیدی را در نظر بگیرید که دارای صفحه ی بادبان خورشیدی دایره ای شکل به شعاع rمی باشد بادبان از جنس آینه ی کامل است.درابتدای قرارگیری فضاپیما در بالای جو در مدار زمین صفحه ی بادبان کاملا در جهت عمود بر خط واصل خورشید با فضاپیما قرار داشت.بعد از آن زاویه ی بین خط عمود بر صفحه و بردار مکان فضاپیما تتا شروع به تغییر کرد.
تتا همان امگا تی (کشتم خودمو با این نوشتنم!!!)است.که امگا خیلی کوچک است.سرعت متوسط دور شدن فضاپیما از خورشید بعد از باز شدن صفحات و دوران آنها را در tصانیه ی کوتاه اول حرکت بیابید!
3.برای سوال 175 مجموعه سوالات آقای چرتاب حس می کنم داده کم داره!اگه این طور نیس میشه توی حل کمکم کنین؟
ممنون!

پیمان اکبرنیا
01-02-2013, 11:34 PM
سلام.منم چند تا سوال دارم.
1.توی سوالات آقای عطا مرادی یک سوال بود که می گفت در یک ستاره که 70 درصد آن هلیوم است و هسته 0.1 شعاع را تشکیل می دهد و دمایش میلیون درجه کلوین است و چگالی ثابت و جرم و شعاع هم مثل خورشیدند.حالا ما باید درصد عناصر سنگین را حساب می کردیم.
من با کمک فرض ثابت بودن چگالی جرم در آن لایه را در آوردم و تعادل هیدرواستاتیک نوشتم و با فرض گاز تماما ایده آل با ساده سازی فشار مرکزی را در آوردم و بعد هم میو را در آنجا در آوردم و با دانستن این که جمع x,zسی درصد است و این که 0.5z,2xهم برآیندشان یک میوام هست مساله رو حل کردم ولی جواب آخر را شک دارم.میشه بچه های دیگه هم جواب آخرشونو بگن چک کنیم؟
2.این سوالاو هنوز به اندازه ی کافی روش فکر نکردم شاید زیاد سخت نباشه ولی گفتم با هم فکر کنیم.
فضاپیمایی خورشیدی را در نظر بگیرید که دارای صفحه ی بادبان خورشیدی دایره ای شکل به شعاع rمی باشد بادبان از جنس آینه ی کامل است.درابتدای قرارگیری فضاپیما در بالای جو در مدار زمین صفحه ی بادبان کاملا در جهت عمود بر خط واصل خورشید با فضاپیما قرار داشت.بعد از آن زاویه ی بین خط عمود بر صفحه و بردار مکان فضاپیما تتا شروع به تغییر کرد.
تتا همان امگا تی (کشتم خودمو با این نوشتنم!!!)است.که امگا خیلی کوچک است.سرعت متوسط دور شدن فضاپیما از خورشید بعد از باز شدن صفحات و دوران آنها را در tصانیه ی کوتاه اول حرکت بیابید!
3.برای سوال 175 مجموعه سوالات آقای چرتاب حس می کنم داده کم داره!اگه این طور نیس میشه توی حل کمکم کنین؟
ممنون!

سلام

سوالات آقای چرتاب در دسترسم نیست ولی در مورد سوال دومی که مطرح کردید.

آیا باید اثر گرانش زمین و خورشید را در نظر بگیریم؟ با توجه به این که اطلاعاتی داده نشده به نظر می آید اینطور نیست و فقط باید اثر فشار تابشی را در نظر بگیریم. باید نیروی فشار تابشی وارد بر این بادبان را حساب کنیم. نکته اینه که زاویه بادبان و راستای عمود بر خورشید عوض می شود. پس هم تعداد فوتونهای دریافتی کم می شود (کاهش سطح مقطع موثر) و هم زاویه تابش و بازتابش از حالت عمودی کم می شود که باعث کمتر شدن تکانه انتقالی می شود. فقط چون زاویه کوچک فرض شده می توان از تقریبهای زوایای کوچک استفاده کرد. حالا شما حل کنید اگر مشکلی بود سوال کنید. اگر حس می کنید راه حلی که میگم درست نیست می تونیم بحث کنیم که چه فرض هایی را اشتباها در نظر نگرفته ام ... :)

بعد از حساب کردن نیرو (تکانه) می توان شتاب و سرعت(از انتگرال شتاب در t ثانیه) را به دست آورد که متغییر هستند و با انتگرال گیری در t ثانیه جابه جایی به دست می آید.

Sara 0i
01-25-2013, 08:51 PM
سلام بر همگی (که نمی شناسم هنوز، کم کم آشنا می شم باهاتون:دی)
اینم یه سوال متوسط مکانیک سماوی اینا تقریبن :
دنباله داری در مدار سهمی حرکت می کند و سرعت آن در حضیض مداریش 60 کیلومتر بر ثانیه اندازه گرفته شده است. در حال حاضر زمین روی محور اصلی سهمی است(به شکل بنگرید!) و فاصله فعلی دنباله دار از خورشید 228 میلیون کیلومتر است و فرض کنید طول دم دنباله 2 میلیون کیلومتر باشد. ناظر روی زمین اندازه زاویه ای دم این دنباله دار را چقدر اندازه می گیرد؟؟(البته که مدار زمین را دایره فرض می کنیم!) 5294

celestial boy
01-25-2013, 09:26 PM
سلام علیکم.
من جوابم 22 دقیقه قوس شد.ولی یه مشکل کلی.دم همونه که در راستای شعاع مداری دنباله دار ایجاد میشه؟؟
اول زاویه تتا واسه اون فاصله رو در آوردم.بعد یه مثلث کشیدمو فاصله دنباله دار از زمینو حسابیدم.بعد از زاویه کوچک حساب کردم.فک نکنم درست باشه؟!!!!!

Sara 0i
01-25-2013, 10:17 PM
خسته نباشید :دی
اما جواب آخر شما صحیح نیست
کمی دقت کنید
فکر می کنم بهتر باشه صبر کنیم تا باز هم آدما فک کنن بعد راهنمایی یا هر بحث کلی ای بکنیم :)

Siavash9874
01-25-2013, 11:03 PM
۱۰.۵ دقیقه قوسی

راه حل : ۲تا مثلثِ مسطحه رو حل می‌کنیم ، اختلاف زاویه‌ها می‌شه جواب.

در یکی‌ فاصله ۲۲۸ می‌شه ، یکی‌ ۳۰۰ .

Sara 0i
01-25-2013, 11:29 PM
شما هم خسته نباشید :)
اگر با دقت 0.1 گزارش کردید باید بگویم خیر، جواب آخر شما هم صحیح نیست
راه حلتان را هم متوجه نشدم ! اختلاف کدام دو زاویه؟ زاویه مربوط به ابتدا و انتهای دم دنباله دار ؟؟

Siavash9874
01-25-2013, 11:54 PM
مثلث اول : زمین ، خورشید ، هسته دنبال در . زاویه زمین خورشید دنبال در ۱۱۰.۴۸ درجه هستش ، زاویه خورشید زمین هسته دنبال در رو پیدا می‌کنیم .

مثلث دوم : زمین ، خورشید ، انتهای دم .

زاویه خورشید زمین هسته :46*35*12.96*

زاویه خورشید زمین دم : 46*45*44.31*

اختلاف ۲ زاویه مقدار زاویه دم می‌شه .

البته می‌شه یه مثلث رو حل کرد که یکی‌ از طول‌ها ۲ میلیون کیلو متر هسته ، یکی‌ ۲۹۴۰ میلیون کیلومتر و یکی‌ دیگه ۲۹۵۷ میلیون کیلومتر.

الان که دوباره حساب کردم مقدار زاویه حدودا ۱۱ دقیقه شد

Sara 0i
01-26-2013, 09:56 AM
صبح بخیر!
البته راه حل شما با چیزی که مد نظر من بود فرق داشت ، ولی جالب بود به نظر درست میاد :)
اما به یک نکته ندارید دقت می کنید که همون احتمالن داره این بلا رو سر جواب میاره

Siavash9874
01-27-2013, 10:58 PM
با اجازه من هم یه سوال بدم :دی

یک منجّم در زمان‌های قدیم در شهری با عرضه جغرافیایی ۳۵ درجه شمالی ، اطلاعات حاصل از رصد ۲ ستاره ۱ و ۲ تا بدین گونه ثبت کرده است :

A۱=۲۸۰ ، a۱=۴۵ ،a۲=۹۰

و همچنین ، این منجّم سمت ستاره آلفا دبّ اصغر ( ستاره قطبی فعلی‌ ) را ۳۲۰ اندازه گرفته است . در حال حاضر ، نظری در همان شهر ستاره ۱ را در سرسو می‌بیند .

محاسبه کنید که منجّم در چند ساله پیش میزیسته است

celestial boy
01-28-2013, 03:41 PM
سلام.
گفتم که اشتباهه.خب بی دلیل از زاویه کوچک استفاده کردم.:دی
بازم می پرسم.این دنباله ای که می فرمایید منظور غباره یا خود دنباله.چون اگه اشتباه نکنم غبار در جهت بردار شعاعیه و دنباله در جهت بردار سرعت.
البته نمی دونم چه ربطی می تونه به جواب داشته باشه ولی....
من از یه راه دیگه هم رفتم.ولی این یکی حتما اشتباه خواهد بود.جواب آخرم 16.2 دقیقه شد.؟؟؟؟؟

Sara 0i
01-28-2013, 03:56 PM
زاویش که واقعن کوچیکه، البته اینی که می گید رو چقدر مطمئنین؟؟ :) آخه دنباله در جهت شعاعه و غبار هم
با توجه به فشار تابشی منحرف می شه دیگه. در جهت سرعت که فکر نکنم راستش باشه
جواب آخر 7 دقیقست

celestial boy
01-28-2013, 04:26 PM
مطمئن که زیاد نیستم.نمی دونم کجا خوندم این مطلبو.ولی یکی ازین دوتا (احتمالا غبار) از بردار شعاعی منحرف میشه و با توجه به مسیر و سرعت در جهت دیگه ای قرار می گیره.حالا این جهت سرعته یا چیز دیگه نمی دونم.(تو پست قبلی اینارو برعکس گفتم!!)

یه چیزی.این همه دقت چجوری ممکنه توی جوابا وجود داشته باشه؟؟همه که نمی تونن دقیقا یک جواب در بیارن.تا چه حد قابل قبوله؟؟

Sara 0i
01-28-2013, 04:54 PM
بله غبار با توجه به فشار تابشی منحرف می شه از راستای شعاعی
آره اکی! ولی دیگه قطعن 2 برابر که در نمیاد
بعدم بعضی از مسائل بازه دارن برای جواب
و خب اگر مثلن ثوابت داده شده باشه اونوخ فقط یه جواب اخر قابل قبوله دیگه :)
اگه جز این باشه که تو این دنیا سنگ رو سنگ بن نمیشه! :دی

shariatzadeh
01-28-2013, 09:04 PM
امشب اومدم که به چالش بکشونمتون . به سوال زیر توجه کنید :

رصدگران ستاره ای قیفاووسی با دوره تناوب 50 روز را با چشمان غیر مسلح رصد کرده اند . بررسی های دقیق این ستاره متغیر ، دو لایه سحابی بازتابی را در اطراف این ستاره نشان میدهد که نور ستاره را بازتاب می کنند . روشنایی هر دو لایه با دوره تناوب 50 روز تغییر می کند . شعاع زاویه ای لایه ها به ترتیب 10 و 21 ثانیه قوس است .
لایه داخلی 30 روز پس از ماکزیمم ستاره متغیر و لایه خارجی 18 روز پس از ماکزیمم ستاره متغیر به ماکزیمم خود می رسند .
فاصله ستاره متغیر از ما چقدر است ؟

celestial boy
01-28-2013, 09:43 PM
سلام.
فک کنم درخشندگی لایه ها باشه.چون بازتابی هستن پس نباید دما تاثیر داشته باشه.و شعاع لایه ها هم.؟؟
ببخشید سوال:الان لایه داخلی روی روشنایی خارجی تاثیر داره دیگه.آیا میشه گفت اختلاف زمان ماکسیمم داخلی و خارجی 38 روزه؟؟یعنی اول لایه داخلی حداکثر میشه.20 روز بعد دوباره متغیر حداکثر و 18 روز بعدش لایه خارجی؟؟؟اصلا چی گفتم!!!!
می دونم خیلی چرت و پرته اگه جواب من 500 تا 600 پارسک باشه.پس نمیگم :دی

shariatzadeh
01-29-2013, 07:22 PM
سلام.
ببخشید سوال:الان لایه داخلی روی روشنایی خارجی تاثیر داره دیگه.آیا میشه گفت اختلاف زمان ماکسیمم داخلی و خارجی 38 روزه؟؟یعنی اول لایه داخلی حداکثر میشه.20 روز بعد دوباره متغیر حداکثر و 18 روز بعدش لایه خارجی؟؟؟اصلا چی گفتم!!!!
می دونم خیلی چرت و پرته اگه جواب من 500 تا 600 پارسک باشه.پس نمیگم :دی

با این شرطی که شما در نظر گرفتید آیا لزومی داره که دوره تناوب روشنایی لایه ها هم 50 روز باشه ؟
شما باید حالت هایی رو پیدا کنید که تمام شرایط توش وجود داشته باشه . بهتره دنبال یه شرط بگردید .
به این نکته هم دقت کنید که ستاره قیفاووسی با چشم غیر مسلح هم رویت شده ، پس فاصله اش نباید خیلی هم زیاد باشه .

المپیاد نجوم
01-30-2013, 10:26 PM
خوب من فکر کردم شاید منظور از 18 روز برای خارجی این باشه که از بیشینه ی قبلی 32 روز بگذره تا این روشن شه وقتی اینم روشن شه مسیر خودش تا اون یکی رو طی می کنه و روشنش می کنه.مگر این که فاصله اش تا اون یکی اون قدر زیاد باشه که خود ستاره ی اون وسط روشن کندش.ولی این طوری انگار یه حالت نمیده!اگرم بده ما در نهایت وقتی اندازه زاویه ای رو داریم و dرو می خواهیم واضحه که باید یه دومعادله دو مجهول در بیاریم .خیلی سوال باحالی به نظر میاد.میشه یکم بیشتر راهنمایی کنین؟

celestial boy
01-30-2013, 10:49 PM
من اینطوری فک کردم.این ماکسیمم واسه لایه داخلی یه جور برایند از نور خود ستاره و نور بازتابی از لایه خارجیه.اینطوری خیلی مسئله پیچیده شد.
راستی الان این لایه ها کروین؟؟یعنی ازون طرف دیگه هم بازتاب میشه روی این طرف؟؟(می دونم خیلی مسخره میشه!!!)اصلا یه شکل کلی از مکان لایه ها راهنمایی کنید.

shariatzadeh
01-31-2013, 08:42 AM
فکر کنم شما یه مشکلی با سحابی بازتابی دارید .
ببینید سحابی بازتابی مثل آینه نیست که نور ستاره رو بازتاب کنه ، بلکه نور ستاره به سحابی بازتابی میرسه ، اون رو گرم میکنه و بعد سحابی مثل یک جسم سیاه تابش میکنه .
توی این سوال هم نور ستاره قیفاووسی بدون هیچ مانعی از تمام لایه ها عبور می کنه و به زمین میرسه ، ولی توی راه لایه ها رو هم گرم میکنه . هیچ نوری از لایه بیرونی به لایه داخلی نمیرسه .

راهنمایی آخر : اگر سرعت نور بینهایت بود ، لایه ها همراه با ماکزیمم ستاره به ماکزیمم خودشون میرسیدن .

المپیاد نجوم
03-08-2013, 05:48 PM
سلام.
من سوال دنباله دار رو 0.58 درجه به دست آوردم.اشتباس؟
اگه اشتباهه من به قسمت به دست آوردن زاویه ی زمین و خورشیدو دنباله دارم شک دارم.لطفا اون بخشو توضیح بدید!
ممنون!

celestial boy
03-08-2013, 08:13 PM
سلام.
یه سوالی هم طلایی های امسال دادن که طول دم دنباله دارو خواسته.این لینک سوالا (http://wdl.persiangig.com/pages/download/?dl=http://ioaa7.persiangig.com/weblog.pdf)
سوال سومش.من راهم خیلی خیلی طولانی شد و آخر هم جوابایی به دست آوردم که با عقل جور در نمیاد!!

راستی سوال آقای شریعت زاده لاینحل موند؟

Laya
03-10-2013, 11:53 PM
سلام. جواب سوال دنباله دار فكر ميكنم به اينصورت باشه، چون سرعت در حضيضو داريم ميتوانيم a در مدار سهمى رو بدست بياريم كه ميشه 10^10*7.41 حالا براى مرحله دوم وقتى r(فاصله تا خورشيد رو داريم ميتونيم زاويه بين راستاى حضيض يا همون مكان زمين در مدارش و خورشيد دنباله دار رو بدست بياريم كه ميشه48 .110درجه. حالا بايد فاصله زمين تا دنباله دار رو بدست بياريم اگر d مكان دنباله دار باشد داريم(ed^2=es^2+sd^2-(2es.sd.cosxو x=110.48است. ed بدست مى ايد 11^10*3.137 حالا دو مثلث داريم يكى esc و ديگرى edc كه c انتهاى دم دنباله دار است در مثلث esc ميتوانيم زاويه روبرو es را بدست بياريم (sc=sd+dc) و dc هم طول دم است زاويه 26.44درجه بدست مي ايد حالا در مثلث edc ضلع edو زاويه روبرويش و ضلع ds را داريم و از رابطه سينوس ها ميتوانيم زاويه روبرويش را كه همان زاويه دم دنباله دار هست رو بدست بياريم كه ميشه 0.162.

المپیاد نجوم
03-14-2013, 01:54 PM
خانم صفری جواب آخرو گفتن و این نیس!بعدا شخصا ازشون خواهم پرسید.
در مورد سوال آقای شریعت زاده من 2 راه به ذهنم می رسه کلا که یکیش داده ای می خواد که توی مساله داده نشده!
1.می دونم مسخره اس ولی گفتم چون ناظر با چشم غیر مسلح داره می بینه پس ته تهش 6.5 می تونه قدرش باشه و قدر خودشم مثلا -5 باشه با توجه به نوعش و ماژول فاصله بنویسیم.که نوشتم در اومد 2 کیلو پارسک.
2.با توجه به راهنمایی که آقای شریعت زاده کردن که البته به ذهن خودمم رسیده بود بیایم از ct=x استفاده کنیم ولی بدون داشتن قطر حلقه ها معادله حل نمیشه. نباید قطر حلقه هارو بدن بهمون؟
کلا اصلا مساله به اینا ربط داره؟اگه نه بازم راهنمایی کنین!لطفا!
خانم صفری گفتن جواب 7 دقیقه قوسه ولی من نمی دونم چرا 70 دقیقه قوس میارم.اون دفعه یه تقسیم بر دو اشتباه کرده بودم و درست کردم ولی بازم 10 برابر در میاد!!!

Laya
03-28-2013, 03:25 PM
سلام فكر كنم اگه اين بخش ها كمى بيشترفعال بشن بد نيست برا همين يه سوال ميزارم.
فرض كنيد قطر زاويه اى ماه ''35 است. لبه پايينى ماه را در عرض جغرافيايى 37درجه در حال غروب بگيريد. ميل مركز ماه 20درجه و '25است. دستگاه مختصات قطبى در مركز قرص ماه قرار مي دهيم. اگر اولين نقطه روى لبه ى ماه كه غروب مي كند داراى زاويه تتاى صفر باشد اخرين نقطه اى كه غروب مي كند چه تتايى دارد؟

celestial boy
04-03-2013, 11:25 AM
سلام فكر كنم اگه اين بخش ها كمى بيشترفعال بشن بد نيست برا همين يه سوال ميزارم.
فرض كنيد قطر زاويه اى ماه ''35 است. لبه پايينى ماه را در عرض جغرافيايى 37درجه در حال غروب بگيريد. ميل مركز ماه 20درجه و '25است. دستگاه مختصات قطبى در مركز قرص ماه قرار مي دهيم. اگر اولين نقطه روى لبه ى ماه كه غروب مي كند داراى زاويه تتاى صفر باشد اخرين نقطه اى كه غروب مي كند چه تتايى دارد؟
سلام.
اینجا باید چرخش وضعی ماه هم در نظر گرفته بشه.نه؟

یه سوال درباره دنباله دارا داشتم..مسیر دم دنباله دار چه ربطی به دمای اون داره؟می تونم از فاصله دنباله دار تا خورشید استفاده کنم بعد شار دریافتی بنویسم؟
ممنون...

Laya
04-03-2013, 11:03 PM
سلام. خير نبايد چرخش وضعى ماه رو در نظر گرفت. ما كره ماه رو روى صفحه در نظر ميگيرم يعنى به شكل دايره. جابجايى ماه در كره سماوي به چرخش وضعى ماه بستگى نداره(همونطور كه ميدونين به علت قفل مدارى ماه تناوب چرخش وضعى ماه با دوره تناوب ماه به دور زمين برابر است و ما همواره يك سمت ماه رو ميبينيم بنابراين اگر دستگاه مختصاتى بر روى ماه قرار دهيم تغييرى نمى كند.)
درمورد سوال دنباله دار، اگه منظورتون از مسير دنباله دار نوع مدار دنباله داره بايد بستگى دما و طول دم يا بستگى فاصله دنباله دار تا جسم مركزى با طول دم را داشته باشيم. و دوباره با مجموع طول دم و فاصله دنباله دار معادله را بدست بياريم. ولى فقط براى دما و فاصله دنباله دار ميتونيم بنويسيم tمتناسب است با0.5-^r.

erfan bayat
04-03-2013, 11:26 PM
پیشنهاد میکنم به عنوان راهنمایی سوال حلقه های زحل (http://erfanbayat.persiangig.com/soal%20astronomy%20olympiad/dore%20tabestan/Picture%20009.jpg)
را ببینید. دلیل چرخش ماه همون دلیل چرخش حلقه هاست

pariya.k
04-09-2013, 08:02 PM
میشه راه حلو بگید ؟:دی ( ببینم درست حلیدم یا نه :دی )

المپیاد نجوم
04-09-2013, 09:14 PM
اون زاویه ای که باید به دست بیاریم 180-sai میشه که sai منظورم همون زاویه ایست که مسیر غروب جسم با افق می سازه که فقط به فی و دلتا ربط داره و دلتای مرکز و کل اندازه رو دارین پس دلتای بالای ماهو دارین و فی همکه داده شده پس حله دیگه زاویه به دست میاد!

pariya.k
04-23-2013, 08:42 PM
اگه میشه این سوالو حل کنید واسم
ناظر a در شهری با عرض جغرافیایی 60 درجه شمالی و طول جغراقیایی 120غربی ماه را بر روی نصف النهار ناظر و با فاصله سمت الرسی 60 درجه رصد میکند . ناظر b در شهری با عرض جغافیایی 30 درجه جنوبی و طول جغرافیایی 120درجه غربی به طور همزمان با ناظر a ماه را بر روی نصف النهار ناظر و با فاصله سمت الراسی 31.28 رصد میکند. فاصله ماه تا مرکز زمین چند برابر شعاع زمین است ؟

erfan bayat
04-24-2013, 09:17 PM
واسه ی حلش دو ناظر رو روی زمین بکش و به ماه وصل کن از مرکز زمین هم به دو ناظر وصل کن که یک چهار ضلعی درست میشه .برای حلش فقط باید یه خط اصافین رسم کنی که از دو تا ناظر به همه. حالا هدف اینه که فاصلهی مرکز زمین تا ماهو بدست بیاری .
زاویه ی زمین ناظر ماه هم همون ارتفاع ماه از دید اون ناظره یا نود منهای فاصله سمت الراسی .زاویه ی بیا دو ناظر از مرکز زمین هم همون اختلاف عرض های دو ناظره چون طولاشاون یکیه. اگه نشد بگو شکلشو بکشم در ضمن از اختلاف منظر ماه صرف نظر کنید

pariya.k
04-26-2013, 11:35 AM
برای حلش فقط باید یه خط اصافین رسم کنی که از دو تا ناظر به همه.

یعنی چی این ؟! الان فاصله دوتا ناظر از هم 90درجست . نمیشه از زمین وصل کنیم به ماه طوری که زاویه 90 درجه رو بکنه دو تا 45 درجه ؟

erfan bayat
04-26-2013, 12:49 PM
یعنی چی این ؟! الان فاصله دوتا ناظر از هم 90درجست . نمیشه از زمین وصل کنیم به ماه طوری که زاویه 90 درجه رو بکنه دو تا 45 درجه ؟
من فاصله ی دو ناظر تا ماهو یکی نگرفتم پس اون زاویه نصف نیست

pariya.k
04-26-2013, 02:18 PM
من فاصله ی دو ناظر تا ماهو یکی نگرفتم پس اون زاویه نصف نیست
آها .خب پس چی کار کنم ؟

arashgmn
05-26-2013, 02:31 PM
آها .خب پس چی کار کنم ؟
کافیه که خط بین مراکز ماه و زمین و همین طور خط های بین مرکز ماه و دوتا ناظر و مرکز زمین و دوتا ناظر ، رسم بشن. دو تا مثلث پدید میاد که (لزوما متشابه و یا هم نهشت نیستند و) یک ضلع هر دوی اون ها برابر با شعاع زمینه. از طرفی دوتا ارتفاع داده شده ، یک زاویه از هر کدوم از این مثلث ها رو به ما میده. تا الآن دو جز از هر مثلث معلوم شده. اگر به اختلاف عرض جغرافیایی دو ناظر هم توجه کنیم ، همه ی اجزای این مثلث ها پیدا می شن و میشه از روی اون ها فاصله زمین تا ماه رو پیدا کرد.

دقت کنید که این سوال ممکنه چندین روش برای حل داشته باشه.

celestial boy
06-01-2013, 05:16 AM
سلام.
در مورد مساحت عرقچین یه سوال دارم.دوتا عرقچین متداخل مساحت مشترکشون چطوری حساب میشه؟
من یه راه حل وحشتناک پیدا کردم.اول توی هندسه مسطح مساحت مشترک دوتا دایره متداخلو حسابیدم.بعد همون راهو تعمیم دادم.ولی دیگه ترسیدم حساب کنم!!!!!

m.Sadat
06-01-2013, 12:44 PM
سلام.
در مورد مساحت عرقچین یه سوال دارم.دوتا عرقچین متداخل مساحت مشترکشون چطوری حساب میشه؟
من یه راه حل وحشتناک پیدا کردم.اول توی هندسه مسطح مساحت مشترک دوتا دایره متداخلو حسابیدم.بعد همون راهو تعمیم دادم.ولی دیگه ترسیدم حساب کنم!!!!!

فکر کنم اگه عرقچین ها رو تصویر کنی روی سطح کره تبدیل بشه به مساحت مشترک بین 2 دایره صغیره که اون موقع راحت حل میشه

celestial boy
06-01-2013, 01:57 PM
سلام.
خب وقتی راحت تر میشه چطوری حل میشه؟! منم همون مساحت مشترک دوتا دایره صغیره گرفتم که یکم سخت شد.میخوام ببینم راه بهتری نیست؟

اصلا راهمو توضیح میدم.دایره های صغیره همو توی دو نقطه قطع می کنن.اونا رو به مرکز دو دایره وصل می کنیم.مثه حالت مسطحه اول مساحت قطاع های ایجاد شده توی دو دایره حساب می کنیم. بعد نقطه های تقاطع رو به هم وصل می کنیم.دوتا مثلث ایجاد میشه (کروی) مساحت هر کدومو از مساحت قطاع مربوطش کم می کنیم.حاصلو با هم جمع می کنیم.
این وسط کلی زاویه باید حساب کنیم.با اینکه فقط جدایی مرکز دایره ها معلومه!!!!

m.Sadat
06-01-2013, 02:24 PM
سلام.
خب وقتی راحت تر میشه چطوری حل میشه؟! منم همون مساحت مشترک دوتا دایره صغیره گرفتم که یکم سخت شد.میخوام ببینم راه بهتری نیست؟

اصلا راهمو توضیح میدم.دایره های صغیره همو توی دو نقطه قطع می کنن.اونا رو به مرکز دو دایره وصل می کنیم.مثه حالت مسطحه اول مساحت قطاع های ایجاد شده توی دو دایره حساب می کنیم. بعد نقطه های تقاطع رو به هم وصل می کنیم.دوتا مثلث ایجاد میشه (کروی) مساحت هر کدومو از مساحت قطاع مربوطش کم می کنیم.حاصلو با هم جمع می کنیم.
این وسط کلی زاویه باید حساب کنیم.با اینکه فقط جدایی مرکز دایره ها معلومه!!!!

این سوالو میتونی مثال زیر بهتر متوجه شی (سوال از آقای عطا مرادی )

دو ماهواره به ترتیب در ارتفاع هاي 2 و 1 برابر شعاع زمین قرار دارند. اگر در یک لحظه جدایی زاویه اي این
دو ماهواره برابر 70 درجه باشد، چند درصد از نواحی کره زمین را به طور مشترك رصد می کنند؟

m.Sadat
06-01-2013, 05:12 PM
این سوالو میتونی مثال زیر بهتر متوجه شی (سوال از آقای عطا مرادی )

دو ماهواره به ترتیب در ارتفاع هاي 2 و 1 برابر شعاع زمین قرار دارند. اگر در یک لحظه جدایی زاویه اي این
دو ماهواره برابر 70 درجه باشد، چند درصد از نواحی کره زمین را به طور مشترك رصد می کنند؟

اینم جواب سوال
http://s2.picofile.com/file/7354094187/%D8%AD%D9%84_%D8%B3%D9%88%D8%A7%D9%84_4.pdf.html
البته اون قسمت آخرش من هرچقدر حساب میکنم رابطه مساحت مشترک اون نمیشه و ضریب 2 برای s1 میاد اگه اون رابطه رو بدست آوردید لطفا راه حلو بگید

المپیاد نجوم
06-01-2013, 07:48 PM
بله اون آخر اشتباه شده منم قبلا فکر می کردم مشکل از منه ولی پرسیدم گفتن اشکال داره!

arashgmn
07-15-2013, 12:09 AM
این سوال برگرفته از یکی از سوال های مجموعه سوالات آقای صدرالدینی است.

در چه زمانی در عرض جغرافیایی 30 درجه شمالی ، همزمان با طلوع خورشید ، ستاره ی سماک رامح غروب می کند و بالعکس ؟!!! مختصات ستاره ی سماک رامح هم در زیر آمده است.

بعد : 14 ساعت و 16 دقیقه و 18 ثانیه ; میل : 19 درجه و 7 دقیقه و 0 ثانیه

m.Sadat
07-15-2013, 11:32 AM
این سوال برگرفته از یکی از سوال های مجموعه سوالات آقای صدرالدینی است.

در چه زمانی در عرض جغرافیایی 30 درجه شمالی ، همزمان با طلوع خورشید ، ستاره ی سماک رامح غروب می کند و بالعکس ؟!!! مختصات ستاره ی سماک رامح هم در زیر آمده است.

بعد : 14 ساعت و 16 دقیقه و 18 ثانیه ; میل : 19 درجه و 7 دقیقه و 0 ثانیه
ممنون از آقا آرش برای طرح این سوال
من حساب کردم و اگه اشتباه نکرده باشم بدست اومد 83 روز بعد از اول بهار یعنی 21 خرداد